Re: [obm-l] Demonstracoes no ensino medio

2004-05-09 Por tôpico Daniel Silva Braz
Bem, na minha faculdade tem essa matéria tb..só que se
chama "Fundamentos de Matemática Elementar"..e é a
mesma coisa..se aprende tudo que se viu no ensino
médio..só que com as demonstrações..realmente é meio
ridiculo isso..já que deveria ser visto deste modo no
segundo grau..mas para se fazer isso é necessário
mudar não só o segundo grau..mas o primeiro tb..e o
curso de licenciatura..Pra se ter uma idéia..existem
pessoas na faculdade (no curso de matemática) que tem
dificuldade em construir/demonstrar provas!! Como vão
fazer para ensinar seus alunos!?!?! E isto não é por
ser uma faculdade particular..estudei durante 2
periodos na UERJ e era a mesma coisa (as vezes até
pior..conheço alunos de lá que não sabem nem q existem
outras geometrias alem da euclidiana!!)..Acho q tem
muita coisa a ser feita para se melhorar o ensino da
matemática..

Daniel S. Braz

=

 --- Johann Peter Gustav Lejeune Dirichlet
<[EMAIL PROTECTED]> escreveu: > So uma
coisa que talvez seja util voces saberem: na
> faculdade a turma de matematica aqui da USP-Sao
> Carlos tem aula de MEB (Matematica do Ensino
> Basico). Curioso, eu perguntei o que sec aprende
> nessa matera e a resposta foi um belo de um " Tudo o
> que se ve no Ensino Medio, com demonstraçoes! ".
> Isto nao e estupido?
>  
> 
> Augusto Cesar de Oliveira Morgado
> <[EMAIL PROTECTED]> wrote:
> Ah, ah, ah!
> Com esses livros didaticos e esses salarios de
> professores, a Matematica no 
> ensino medio nao tem mais teoremas, tem, no maximo,
> observações.
> O Provão mostrou que cerca de 20% dos licenciandos
> em Matemática (aí 
> incluidos os formandos de universidades
> conceituadissimas) não sabem deduzir 
> a fórmula que resolve equação do segundo grau.
> Morgado
> 
>
==
> Mensagem enviada pelo CIP WebMAIL - Nova Geração -
> v. 2.1
> CentroIn Internet Provider
> http://www.centroin.com.br
> Tel: (21) 2542-4849, (21) 2295-3331 Fax: (21)
> 2295-2978
> Empresa 100% Brasileira - Desde 1992 prestando
> servicos online
> 
> 
> -- Original Message ---
> From: Claudio Buffara 
> To: Lista OBM 
> Sent: Sun, 09 May 2004 21:43:51 -0300
> Subject: [obm-l] Demonstracoes no ensino medio
> 
> > Esta eh uma pergunta para quem estah cursando ou
> lecionando 
> > matematica no ensino medio "normal" (ou seja,
> excluindo cursos de 
> > preparacao para olimpiadas ou vestibulares muito
> puxados como IME e 
> > ITA):
> > 
> > Em provas e exercicios propostos exige-se que os
> alunos apresentem
> > demonstracoes de resultados matematicos ou apenas
> pede-se que eles calculem
> > coisas?
> > 
> > []s,
> > Claudio.
> > 
> >
>
=
> > Instruções para entrar na lista, sair da lista e
> usar a lista em
> >
> http://www.mat.puc-rio.br/~nicolau/olimp/obm-l.html
> >
>
=
> --- End of Original Message ---
> 
>
=
> Instruções para entrar na lista, sair da lista e
> usar a lista em
> http://www.mat.puc-rio.br/~nicolau/olimp/obm-l.html
>
=
> 
> TRANSIRE SVVM PECTVS MVNDOQVE POTIRI 
> 
> CONGREGATI EX TOTO ORBE MATHEMATICI OB SCRIPTA
> INSIGNIA TRIBVERE 
> 
> Fields Medal(John Charles Fields)
>  
> N.F.C. (Ne Fronti Crede)
> 
> 
> 
> 
> -
> Yahoo! Messenger - Fale com seus amigos online.
> Instale agora! 

__

Yahoo! Messenger - Fale com seus amigos online. Instale agora! 
http://br.download.yahoo.com/messenger/
=
Instruções para entrar na lista, sair da lista e usar a lista em
http://www.mat.puc-rio.br/~nicolau/olimp/obm-l.html
=


[obm-l] Re: [obm-l] Demonstração

2004-05-09 Por tôpico tyum


-- Mensagem original --

>Olá
>
>Qual a maneira correta e mais lógica de fazer uma demonstração:
>
>Para a e b em R quaisquer prove que a(-b) = -(ab) = (-a)b
>
>
>Vamos lá:

>Sabemos que b+(-b)=0 (axioma)
Assim, a[b+(-b)]=a.0=>(distributiva) a.b+a.(-b)=0
-(a.b)+a.b+a.(-b)=-(a.b)+0=>a.(-b)=-(a.b)

Douglas
>--
>Everton Antonio Ramos (44) 8801-0186
>[EMAIL PROTECTED]
>
>Av. Dr. Luiz Teixeira Mendes, 638
>Maringá - Paraná
>(44) 3028-6300
>--
>
>- Original Message - 
>From: "Augusto Cesar de Oliveira Morgado" <[EMAIL PROTECTED]>
>To: <[EMAIL PROTECTED]>
>Sent: Sunday, May 09, 2004 10:53 PM
>Subject: Re: [obm-l] 8a cone sul
>
>
>> 10a+b-a^2-b^2 = 25,25 -(a-5)^2 - (b-0,5)^2
>> a = 5 e b=0 ou b=1
>> ==
>> Mensagem  enviada  pelo  CIP  WebMAIL  - Nova Geração - v. 2.1
>> CentroIn Internet Provider  http://www.centroin.com.br
>> Tel: (21) 2542-4849, (21) 2295-3331Fax: (21) 2295-2978
>> Empresa 100% Brasileira - Desde 1992 prestando servicos online
>>
>>
>> -- Original Message ---
>> From: [EMAIL PROTECTED]
>> To: [EMAIL PROTECTED]
>> Sent: Sun, 9 May 2004 21:43:41 -0300
>> Subject: [obm-l] 8a cone sul
>>
>> > >>Por favor alguém tem idea de como posso resolver esse problema. De
>cada
>> > número inteiro positivo n,n<=99,subtraímos a soma dos quadrados de
>> > seus algarismos.Para que valores de n esta diferença é a maior
>> > possível?
>> >
>> > _
>> > Voce quer um iGMail protegido contra vírus e spams?
>> > Clique aqui: http://www.igmailseguro.ig.com.br
>> > Ofertas imperdíveis! Link: http://www.americanas.com.br/ig/
>> >
>> >
>=
>> > Instruções para entrar na lista, sair da lista e usar a lista em
>> > http://www.mat.puc-rio.br/~nicolau/olimp/obm-l.html
>> >
>=
>> --- End of Original Message ---
>>
>> =
>> Instruções para entrar na lista, sair da lista e usar a lista em
>> http://www.mat.puc-rio.br/~nicolau/olimp/obm-l.html
>> =
>>
>
>
>=
>Instruções para entrar na lista, sair da lista e usar a lista em
>http://www.mat.puc-rio.br/~nicolau/olimp/obm-l.html
>=
>



--
Use o melhor sistema de busca da Internet
Radar UOL - http://www.radaruol.com.br




=
Instruções para entrar na lista, sair da lista e usar a lista em
http://www.mat.puc-rio.br/~nicolau/olimp/obm-l.html
=


Re: [obm-l] O que eh "AO ACASO"?

2004-05-09 Por tôpico Gabriel Reina
É isso aí, Buffas!
\o/  \o/  \o/  \o/

Mas afinal... O que é "AO ACASO"? Hein?
§:8-)

-- Gabriel

- Original Message - 
From: "Claudio Buffara" <[EMAIL PROTECTED]>
Subject: Re: [obm-l] O que eh "AO ACASO"?


> Esse Gabriel reclamou bastante da lista (ateh com alguma razao) mas ainda
> nao fez nenhuma contribuicao digna de nota.

=
Instruções para entrar na lista, sair da lista e usar a lista em
http://www.mat.puc-rio.br/~nicolau/olimp/obm-l.html
=


Re: [obm-l] Demonstracoes no ensino medio

2004-05-09 Por tôpico Augusto Cesar de Oliveira Morgado



E a USP Sao Carlos é conceituadissima!

== 
Mensagem  enviada  pelo  CIP  WebMAIL  - Nova Geração - v. 2.1 
CentroIn Internet Provider          http://www.centroin.com.br 
Tel: (21) 2542-4849, (21) 2295-3331        Fax: (21) 2295-2978 
Empresa 100% Brasileira - Desde 1992 prestando servicos online 


-- Original Message ---
From: Johann Peter Gustav Lejeune Dirichlet <[EMAIL PROTECTED]> 
To: [EMAIL PROTECTED] 
Sent: Sun, 9 May 2004 23:01:26 -0300 (ART) 
Subject: Re: [obm-l] Demonstracoes no ensino medio 

> So uma coisa que talvez seja util voces saberem: na faculdade a turma de matematica aqui da USP-Sao Carlos tem aula de MEB (Matematica do Ensino Basico). Curioso, eu perguntei o que sec aprende nessa matera e a resposta foi um belo de um " Tudo o que se ve no Ensino Medio, com demonstraçoes! ". 
> Isto nao e estupido? 
>   
> 
> Augusto Cesar de Oliveira Morgado <[EMAIL PROTECTED]> wrote: Ah, ah, ah!
> Com esses livros didaticos e esses salarios de professores, a Matematica no 
> ensino medio nao tem mais teoremas, tem, no maximo, observações.
> O Provão mostrou que cerca de 20% dos licenciandos em Matemática (aí 
> incluidos os formandos de universidades conceituadissimas) não sabem deduzir 
> a fórmula que resolve equação do segundo grau.
> Morgado
> 
> ==
> Mensagem enviada pelo CIP WebMAIL - Nova Geração - v. 2.1
> CentroIn Internet Provider http://www.centroin.com.br
> Tel: (21) 2542-4849, (21) 2295-3331 Fax: (21) 2295-2978
> Empresa 100% Brasileira - Desde 1992 prestando servicos online
> 
> -- Original Message ---
> From: Claudio Buffara <[EMAIL PROTECTED] />
> To: Lista OBM <[EMAIL PROTECTED] />
> Sent: Sun, 09 May 2004 21:43:51 -0300
> Subject: ! [obm-l] Demonstracoes no ensino medio
> 
> > Esta eh uma pergunta para quem estah cursando ou lecionando 
> > matematica no ensino medio "normal" (ou seja, excluindo cursos de 
> > preparacao para olimpiadas ou vestibulares muito puxados como IME e 
> > ITA):
> > 
> > Em provas e exercicios propostos exige-se que os alunos apresentem
> > demonstracoes de resultados matematicos ou apenas pede-se que eles calculem
> > coisas?
> > 
> > []s,
> > Claudio.
> > 
> > =
> > Instruções para entrar na lista, sair da lista e usar a lista em
> > http://www.mat.puc-rio.br/~nicolau/olimp/obm-l.html
> > =
> --- End of Original Message ---
> 
> =
> Instruções para entrar na lista, sair da lista e usar a lista em
> http://www.mat.puc-rio.br/~nicolau/olimp/obm-l.html
> =
> 
> TRANSIRE SVVM PECTVS MVNDOQVE POTIRI 
> 
> CONGREGATI EX TOTO ORBE MATHEMATICI OB SCRIPTA INSIGNIA TRIBVERE 
> 
> Fields Medal(John Charles Fields) 
>   
> N.F.C. (Ne Fronti Crede)
> 
> 
Yahoo! Messenger - Fale com seus amigos online. Instale agora! 
--- End of Original Message ---






Re: [obm-l] Demonstracoes no ensino medio

2004-05-09 Por tôpico Johann Peter Gustav Lejeune Dirichlet
So uma coisa que talvez seja util voces saberem: na faculdade a turma de matematica aqui da USP-Sao Carlos tem aula de MEB (Matematica do Ensino Basico). Curioso, eu perguntei o que sec aprende nessa matera e a resposta foi um belo de um " Tudo o que se ve no Ensino Medio, com demonstraçoes! ".
Isto nao e estupido?
 
Augusto Cesar de Oliveira Morgado <[EMAIL PROTECTED]> wrote:
Ah, ah, ah!Com esses livros didaticos e esses salarios de professores, a Matematica no ensino medio nao tem mais teoremas, tem, no maximo, observações.O Provão mostrou que cerca de 20% dos licenciandos em Matemática (aí incluidos os formandos de universidades conceituadissimas) não sabem deduzir a fórmula que resolve equação do segundo grau.Morgado==Mensagem enviada pelo CIP WebMAIL - Nova Geração - v. 2.1CentroIn Internet Provider http://www.centroin.com.brTel: (21) 2542-4849, (21) 2295-3331 Fax: (21) 2295-2978Empresa 100% Brasileira - Desde 1992 prestando servicos online-- Original Message ---From: Claudio Buffara <[EMAIL PROTECTED]>To: Lista OBM <[EMAIL PROTECTED]>Sent: Sun, 09 May 2004 21:43:51 -0300Subject: [obm-l]
 Demonstracoes no ensino medio> Esta eh uma pergunta para quem estah cursando ou lecionando > matematica no ensino medio "normal" (ou seja, excluindo cursos de > preparacao para olimpiadas ou vestibulares muito puxados como IME e > ITA):> > Em provas e exercicios propostos exige-se que os alunos apresentem> demonstracoes de resultados matematicos ou apenas pede-se que eles calculem> coisas?> > []s,> Claudio.> > => Instruções para entrar na lista, sair da lista e usar a lista em> http://www.mat.puc-rio.br/~nicolau/olimp/obm-l.html> =--- End of Original Message ---=Innstruções para entrar na lista, sair da lista e usar a lista
 emhttp://www.mat.puc-rio.br/~nicolau/olimp/obm-l.html=
TRANSIRE SVVM PECTVS MVNDOQVE POTIRI CONGREGATI EX TOTO ORBE MATHEMATICI OB SCRIPTA INSIGNIA TRIBVERE Fields Medal(John Charles Fields)
 
N.F.C. (Ne Fronti Crede)Yahoo! Messenger - Fale com seus amigos online. Instale agora!

Re: [obm-l] RE: [obm-l] Princípio de Dirichlet - variacao

2004-05-09 Por tôpico Ricardo Bittencourt
Frederico Reis Marques de Brito wrote:
Considere o conjunto S dos pontos do R^2 que distam, na métrica 
euclidiana, 1 unidade da origem do R^2. Se a cada ponto de S  
associarmos um elemento do conjunto  T={A,B} então existirão sempre três 
pontos de S equidistantes ( na métrica euclidiana ) associados a um 
mesmo elemento de T.
Mas é verdade isso mesmo? Sejam dois pontos A,B;
então o lugar geométrico dos pontos distantes do ponto A
um comprimento d(AB) é um círculo de raio d(AB) centrado
em A, o mesmo vale pra B. Os dois círculos se encontram
em dois pontos, que determinam as duas únicas possíveis
posições para um ponto C tal que os três sejam equidistantes,
e nessas condições ABC formam um triângulo equilátero.
Agora, se pra resolver o problema você precisa
inscrever um triângulo equilátero no seu conjunto S,
então vai dar zica. Quebre o conjunto S em três intervalos
semi-abertos R=[0,120[ , S=[120,240[ , T=[240,360[
(ângulos em graus). Para um triângulo equilátero estar
inscrito no conjunto S, precisa ter um ponto em cada
um desses intervalos. Mas agora eu pinto de azul os
conjuntos R e S, e de vermelho o conjunto T, e garanto
que não há triângulos equiláteros com vértices de mesma cor.

Ricardo Bittencourt   http://www.mundobizarro.tk
[EMAIL PROTECTED]   "tenki ga ii kara sanpo shimashou"
-- União contra o forward - crie suas proprias piadas --
=
Instruções para entrar na lista, sair da lista e usar a lista em
http://www.mat.puc-rio.br/~nicolau/olimp/obm-l.html
=


Re: [obm-l] 8a cone sul

2004-05-09 Por tôpico Johann Peter Gustav Lejeune Dirichlet
Essa e simples mas divertida.
Se n e um natural de dois algarismos entao n=10a+b, a e b de 0 a 9, a<>0.
Assim sendo, a subtraçao vai dar 10a+b-a^2-b^2=a(10-a)+b(1-b)
E facil (use Medias) provar que isto e <=25+0 e que isso so da certo se n=50 ou 51.[EMAIL PROTECTED] wrote:
>>Por favor alguém tem idea de como posso resolver esse problema. De cada número inteiro positivo n,n<=99,subtraímos a soma dos quadrados de seus algarismos.Para que valores de n esta diferença é a maior possível? _Voce quer um iGMail protegido contra vírus e spams? Clique aqui: http://www.igmailseguro.ig.com.brOfertas imperdíveis! Link: http://www.americanas.com.br/ig/=Instruções para entrar na lista, sair da lista e usar a lista emhttp://www.mat.puc-rio.br/~nicolau/olimp/obm-l.html=
TRANSIRE SVVM PECTVS MVNDOQVE POTIRI CONGREGATI EX TOTO ORBE MATHEMATICI OB SCRIPTA INSIGNIA TRIBVERE Fields Medal(John Charles Fields)
 
N.F.C. (Ne Fronti Crede)Yahoo! Messenger - Fale com seus amigos online. Instale agora!

Re: [obm-l] Demonstração

2004-05-09 Por tôpico niski
Depende com quem voce esta falando.
Se for com o publico do ensino medio nao tao forte, uma "prova" 
geometrica é suficiente.
Se for com o publico do ensino medio forte, uma demonstracao mais formal 
como a apresentada nos livros de analise do Elon é suficiente. Em todo 
caso, sou da opiniao que a "prova" geometrica nunca deve ser omitida.


Qual a maneira correta e mais lógica de fazer uma demonstração:
Para a e b em R quaisquer prove que a(-b) = -(ab) = (-a)b

--
Niski - http://www.linux.ime.usp.br/~niski
[upon losing the use of his right eye]
"Now I will have less distraction"
Leonhard Euler
=
Instruções para entrar na lista, sair da lista e usar a lista em
http://www.mat.puc-rio.br/~nicolau/olimp/obm-l.html
=


Re: [obm-l] Dominos e Fibonacci (e sapinhos na escada!)

2004-05-09 Por tôpico Johann Peter Gustav Lejeune Dirichlet
Numero de Fibonacci.
Este problema foi discutido na Lista ha algum tempo, e o Shine falou que obteve uma contagem dupla bem interessante...
Seja f(n) o numero de modos pedido no enunciado.
Pegue um tabuleiro 2x(n+2).Imagine-o copmo normalmente voce imaginaria:
|_|_|_|_|_|_|_|_|_|_|_|_|_|_|_|_|_|_|_|_|_|_|_|_|_|_|_|
|_|_|_|_|_|_|_|_|_|_|_|_|_|_|_|_|_|_|_|_|_|_|_|_|_|_|_|
 
(eu nao presto pra artista ASCII)
 
Vamos provar que f(n+2)=f(n+1)+f(n).
Olhando as duas ultimas colunas, temos os casos a seguir
1)Dois dominos deitados.
Ai o restante e coberto de f(n) modos.
2)Um domino de pe
Ai o resto e coberto de f(n+1) modos.
 
Como estes casos sao todos os que se pode testar, f(n+2)=f(n+1)+f(n). E acabou!
 
Um problema correlato: 
"Um sapo está na base de uma escada.Suas pernas permitem que ele pule somente 1 ou 2 andares (imagina so o tamanho do degrau...). Sabendo que ele nao desce nunca a escada, de quantos modos ele chega ate o enesimo degrau?"
Se quiser tente ver uma bijeçao entre esses dois problemas!
 
Te mais!
Ass.:JohannClaudio Buffara <[EMAIL PROTECTED]> wrote:
De quantas maneiras podemos cobrir um tabuleiro 2xn com dominos?Suponha que os dominos sao simetricos (ou seja, ambos os quadrados tem omesmo numero de bolinhas).=Instruções para entrar na lista, sair da lista e usar a lista emhttp://www.mat.puc-rio.br/~nicolau/olimp/obm-l.html=
TRANSIRE SVVM PECTVS MVNDOQVE POTIRI CONGREGATI EX TOTO ORBE MATHEMATICI OB SCRIPTA INSIGNIA TRIBVERE Fields Medal(John Charles Fields)
 
N.F.C. (Ne Fronti Crede)Yahoo! Messenger - Fale com seus amigos online. Instale agora!

Re: [obm-l] 8a.cone sul

2004-05-09 Por tôpico Johann Peter Gustav Lejeune Dirichlet
Sim, eu vou fazer algo mais construtivo.
 
Va a Eureka! que la tem um otimo artigo sobre como resolver equaçoes de Pell-Fermat. O Caminha (o cara que fez o artigo) explica isso la beeem melhor que eu.
E eu nao expliquei a todos pois imaginava que voces ja soubessem o que e que e.Caso alguem nao saiba,Equaçao de Pell-Fermat e toda equaçao diofantina (eq. diofantina e aquela com variaveis em Z) da forma x^2=d*y^2+1(com d nao-quadrado).
E essa minha soluçao foi meio magica mesmo.Quando eu estava tentando achar alguns polinomios que davam certo, saiu isso ai...Tentei pensar em Pitagoras e coisa e tal, mas nao saia nada!Ai resolvi escrever a,b,c como polinomoios de grau 2 em t e comecei a tentar ver algumas variaveis...Mas nada que valha mesmo a pena gastar dedo digitando.
E as vezes meu tempo e bem pequeno cara, eu nem sempre posso postar soluçoes completas.Se eu nao fosse tao perfeccionista e lerdo na digitaçao poderia ate valer de algo...
Alias voce nao pode ter pena de alguem sendo que ainda nao tenho alunos (nem sei se terei) e nem esta na pele de um deles...Mas tudo bem, assim e a vida, aprender com as proprias topadas.
Te mais!!!Ass.:Johann
PS.:Sera que existe um metodo para obter o conjunto de soluçoes disso ai?
 
 
Fabio Henrique <[EMAIL PROTECTED]> wrote:
Cara, não leva a mal. Você continua não colaborando com coisa alguma. Ou você faz observações que não acrescentam nada, ou faz este tipo de intervenção mágica. Isto é pouco instrutivo. O que são as equações de Pell? Por que você não explica para todos? Eu não sei o que são estas equações e resolvi. Acho que você é o maior enrolão. Pare de embromar e faça algo de construtivo como o Buffara, Steiner, Morgado, Nicolau, Santa Rita, Cyberhelp, etc. Tenho pena de quem vir a ser seu aluno. Passe bem. Em 8 May 2004, [EMAIL PROTECTED] escreveu: >a=15*t^2-15t-995 >b=15*t^2-20t-992 >c=15*t^2-18t-993 > > --- Johann Peter Gustav Lejeune Dirichlet > escreveu: > E >so usar umas equaçoes de Pell e o problema >t;> sai. >> A verdadeira treta ai e achar Todas e somente
 >> TODAS as soluçoes. Encara essa! >> >> --- [EMAIL PROTECTED] escreveu: > Essa é >> boa.Demonstrar que existem infinitos >> > ternos (a,b,,c),com a,b,c números >> > naturais,que satisfazem a >> > relação:2a^2+3b^2-5c^2=1997.Abraços vieira. >> > >> > >> >_ >> > Voce quer um iGMail protegido contra vírus e >> > spams? >> > Clique aqui: >> http://www.igmailseguro.ig.com.br >> > Ofertas imperdíveis! Link: >> > http://www.americanas.com.br/ig/ >> > >> > >> >= >> > Instruções para entrar na lista, sair da >> lista >> > e usar a lista em >> > >>
 >http://www.mat.puc-rio.br/~nicolau/olimp/obm-l.html >> > >> >= >> >> >> = >> TRANSIRE SVVM PECTVS MVNDOQVE POTIRI >> >> CONGREGATI EX TOTO ORBE MATHEMATICI OB SCRIPTA >> INSIGNIA TRIBVERE >> >> Fields Medal(John Charles Fields) >> >> N.F.C. (Ne Fronti Crede) >> >> >> >> >__ >> >> Yahoo! Messenger - Fale com seus amigos online. >> Instale agora! >> http://br.download.yahoo.com/messenger/ >> >= >> Instruções para entrar na lista, sair da lista >> e usar a lista em >>
 >http://www.mat.puc-rio.br/~nicolau/olimp/obm-l.html >> >= > >= >TRANSIRE SVVM PECTVS MVNDOQVE POTIRI > >CONGREGATI EX TOTO ORBE MATHEMATICI OB SCRIPTA INSIGNIA TRIBVERE > >Fields Medal(John Charles Fields) > >N.F.C. (Ne Fronti Crede) > >__ > >Yahoo! Messenger - Fale com seus amigos online. Instale agora! >http://br.download.yahoo.com/messenger/ >= >Instruções para entrar na lista, sair da lista e usar a lista em >http://www.mat.puc-rio.br/~nicolau/olimp/obm-l.html >= > >--
 _Voce quer um iGMail protegido contra vírus e spams? Clique aqui: http://www.igmailseguro.ig.com.br
TRANSIRE SVVM PECTVS MVNDOQVE POTIRI CONGREGATI EX TOTO ORBE MATHEMATICI OB SCRIPTA INSIGNIA TRIBVERE Fields Medal(John Charles Fields)
 
N.F.C. (Ne Fronti Crede)Yahoo! Messenger - Fale com seus amigos online. Instale agora!

Re: [obm-l] Demonstração

2004-05-09 Por tôpico Claudio Buffara
on 09.05.04 22:01, Everton A. Ramos (www.bs2.com.br) at [EMAIL PROTECTED]
wrote:

> Olá
> 
> Qual a maneira correta e mais lógica de fazer uma demonstração:
> 
> Para a e b em R quaisquer prove que a(-b) = -(ab) = (-a)b
> 
> 
Primeiro prove que a*0 = 0*a = 0, para todo a em R.
Depois, use a propriedade distributiva:
0 = a*0 = a*(b + (-b)) = a*b + a*(-b) ==> a*-b) = -(a*b)

[]s,
Claudio.


=
Instruções para entrar na lista, sair da lista e usar a lista em
http://www.mat.puc-rio.br/~nicolau/olimp/obm-l.html
=


Re: [obm-l] 8a cone sul

2004-05-09 Por tôpico Claudio Buffara
on 09.05.04 21:43, [EMAIL PROTECTED] at [EMAIL PROTECTED] wrote:

>>> Por favor alguém tem idea de como posso resolver esse problema. De cada
> número inteiro positivo n,n<=99,subtraímos a soma dos quadrados de seus
> algarismos.Para que valores de n esta diferença é a maior possível?
> 
O problema eh maximizar D = 10a + b - a^2 - b^2, onde:
1 <= a <= 9   e   0 <= b <= 9  (a,b: inteiros)

D = a(10 - a) - b(b - 1)

Para maximizar D, devemos maximizar a(10 - a) e minimizar b(b - 1).

b(b - 1) eh sempre nao-negativo.
Logo, eh minimo e igual a 0 quando b = 0 ou b = 1

a(10 - a) eh maximo e igual a 25 quando a = 5 (propriedade elementar do
trinomio do 2o. grau).

Logo, o valor maximo de D eh 25 e ocorre para n = 50 ou 51.

*

Pergunta: Quanto tempo voce passa tentando resolver cada problema desses que
voce tem mandado pra lista?


[]s,
Claudio.


=
Instruções para entrar na lista, sair da lista e usar a lista em
http://www.mat.puc-rio.br/~nicolau/olimp/obm-l.html
=


Re: [obm-l] Demonstracoes no ensino medio

2004-05-09 Por tôpico Fellipe Rossi
a maioria, no meu modo de ver, exige cálculos.
Isso resulta em, quando pedem-se demonstrações, mesmo as mais simples, serem
questões consideradas difíceis. Até mesmo para questões literais os
vestibulando encontram dificuldades.

Aproveitando que fez referência sobre IME/ITA, vou propor uma questão que vi
em uma lista preparatoria e achei interessante:


**
Tome no plano xOy, A(a,0) e B(o,b)
Sejam A'(a+d,0) e B' (0,b+d), d Real.

Mostre que todas as mediatrizes dos segmentos A'B' Se interceptam em um
ponto.
**

Rossi


- Original Message -
From: "Claudio Buffara" <[EMAIL PROTECTED]>
To: "Lista OBM" <[EMAIL PROTECTED]>
Sent: Sunday, May 09, 2004 9:43 PM
Subject: [obm-l] Demonstracoes no ensino medio


> Esta eh uma pergunta para quem estah cursando ou lecionando matematica no
> ensino medio "normal" (ou seja, excluindo cursos de preparacao para
> olimpiadas ou vestibulares muito puxados como IME e ITA):
>
> Em provas e exercicios propostos exige-se que os alunos apresentem
> demonstracoes de resultados matematicos ou apenas pede-se que eles
calculem
> coisas?
>
> []s,
> Claudio.
>
> =
> Instruções para entrar na lista, sair da lista e usar a lista em
> http://www.mat.puc-rio.br/~nicolau/olimp/obm-l.html
> =
>
>


=
Instruções para entrar na lista, sair da lista e usar a lista em
http://www.mat.puc-rio.br/~nicolau/olimp/obm-l.html
=


Re: [obm-l] Re: [obm-l] Re: [obm-l] Questão de 2o. grau

2004-05-09 Por tôpico Augusto Cesar de Oliveira Morgado



A sugestão do Marcio Cohen parece-me a melhor maneira de resolver a questão para alunos do ensino medio.
Para n>4, n! é multiplo de 10, ou seja, termina em 0. Logo, nao ha alteraçao do digito final da soma para n>4.
Para n=1, a soma vale 1; para n=2, vale 3; para n=3, vale9; para n=4, vale 33; a partir daí, todos os valores terminam em 3 e nao ha mais quadrados perfeitos.

== 
Mensagem  enviada  pelo  CIP  WebMAIL  - Nova Geração - v. 2.1 
CentroIn Internet Provider          http://www.centroin.com.br 
Tel: (21) 2542-4849, (21) 2295-3331        Fax: (21) 2295-2978 
Empresa 100% Brasileira - Desde 1992 prestando servicos online 


-- Original Message ---
From: "Fellipe Rossi" <[EMAIL PROTECTED]> 
To: <[EMAIL PROTECTED]> 
Sent: Sun, 9 May 2004 21:11:54 -0300 
Subject: [obm-l] Re: [obm-l] Re: [obm-l] Questão de 2o. grau 

> Sim, mas essas demonstrações exigem uma experiência raramente encontrada em alunos de 2o. grau. 
>   
> Outra questão do tipo seria mostrar que pra n>1, n pertence a N, que (n^2)! > (n!)^2 
>   
> No caso eu faria: 
>   
> (n^2)! = 1*2*3*...*n^2 = 1*2*3...*n*(n+1)*(n+2)*...*n^2 
> (n!)^2= 1*1*2*2*3*3*4*4*...*n*n = 1*2*3...*n*1*2*3...*n 
>   
> como (n+1)(n+2)...n^2 > 1*2*3...*n    pois n+1>1, n+2>2,..., n^2>n provamos 
>   
> Porém é mais um tipo de resolução que raramente entra na cabeça de vestibulandos. 
>   
> Oq é mais normal fazer seria: 
> (2^2)! = 24 
> (2!)^2 = 4 
>   
> (3^2)! = 9! 
> (3!)^2 = 36 
>   
> para valores maiores de n maior será a diferença... 
>   
> O que eu queria saber é se uma Banca de vestibular aceita esses tipos de resolução. 
>   
> Grato pela atenção! 
> Rossi 
> - Original Message - 
> From: Marcio Afonso A. Cohen 
> To: [EMAIL PROTECTED] 
> Sent: Sunday, May 09, 2004 6:14 PM 
> Subject: [obm-l] Re: [obm-l] Questão de 2o. grau 
> 
>     Em parte. Tudo que voce diz eh verdade, mas eu exigiria uma explicacao um pouquinho melhor de pq n! eh maior que n^2. Mas a ideia eh otima e funciona. Eu acho q faria algo como: p/ n>3, 1! + 2! + ... + n! >= n! + (n-1)!+1 > n(n-1) + (n-1) + 1 = n^2. 
>     Uma outra opcao eh olhar mod 10. 
> - Original Message - 
> From: Fellipe Rossi 
> To: [EMAIL PROTECTED] 
> Sent: Sunday, May 09, 2004 5:42 PM 
> Subject: [obm-l] Questão de 2o. grau 
> 
> Como vocês demonstrariam, para 2o. grau, que 
>   
> para n>=1, n pertence a Z. apenas n=1 e n=3 são raízes da equação: 
>   
> 1!+2!+3!+...+n! = n^2 
>   
> Vocês aceitariam uma resolução que mostrasse, com exemplos (4!=24, 4^2=16 ; 5!=120, 5^2=25, e assim por diante...) que para n>=4. n! é maior que n^2 e que como o lado esquerdo da igualdade eh n!+valor positivo, ela vai ser sempre maior que o lado direito para n>=4, e substituindo n por 1, 2 e 3 chegamos q apenas 1 e 3 são raizes? 
>   
> Essa qustão caiu, se não me engano, na prova específica da UFRJ 1992. 
>   
> Abraços! 
> Rossi


--- End of Original Message ---






Re: [obm-l] Questão_de_2o._grau

2004-05-09 Por tôpico Johann Peter Gustav Lejeune Dirichlet
Eu nao daria pontuaçao integral, mas essa ja merece alguma coisa...
Voce poderia falar algo como
 "O lado direito cresce mais rapidamente que o esquerdo, de modo que alguma hora um deles superara o outro.Como isto ocorre para n=4, dfevera ocorrer para n>=4."Fellipe Rossi <[EMAIL PROTECTED]> wrote:




Como vocês demonstrariam, para 2o. grau, que 
 
para n>=1, n pertence a Z. apenas n=1 e n=3 são raízes da equação:
 
1!+2!+3!+...+n! = n^2 
 
Vocês aceitariam uma resolução que mostrasse, com exemplos (4!=24, 4^2=16 ; 5!=120, 5^2=25, e assim por diante...) que para n>=4. n! é maior que n^2 e que como o lado esquerdo da igualdade eh n!+valor positivo, ela vai ser sempre maior que o lado direito para n>=4, e substituindo n por 1, 2 e 3 chegamos q apenas 1 e 3 são raizes?
 
Essa qustão caiu, se não me engano, na prova específica da UFRJ 1992.
 
Abraços! 
Rossi
TRANSIRE SVVM PECTVS MVNDOQVE POTIRI CONGREGATI EX TOTO ORBE MATHEMATICI OB SCRIPTA INSIGNIA TRIBVERE Fields Medal(John Charles Fields)
 
N.F.C. (Ne Fronti Crede)Yahoo! Messenger - Fale com seus amigos online. Instale agora!

Re: [obm-l] 8a.cone sul

2004-05-09 Por tôpico Fabio Henrique
O recado foi para o Dirichlet. 


Em 9 May 2004, [EMAIL PROTECTED] escreveu: 

>Em 9 May 2004, [EMAIL PROTECTED] escreveu: 
> 
>>Cara, não leva a mal. 
>>Você continua não colaborando com coisa alguma. Ou você faz observações 
que 
>>não acrescentam nada, ou faz este tipo de intervenção mágica. Isto é pouco 
>>instrutivo. O que são as equações de Pell? Por que você não explica para 
>>todos? Eu não sei o que são estas equações e resolvi. Acho que você é o 
>>maior enrolão. Pare de embromar e faça algo de construtivo como o Buffara, 
>>Steiner, Morgado, Nicolau, Santa Rita, Cyberhelp, etc. 
>>Tenho pena de quem vir a ser seu aluno. 
>>Passe bem. 
> 
>>Não é nada disso, sou muito esforçado e estou tentando aprender, se voçê 
>não pode me ajudar não me desanime.Pena eu tenho de voçê por não me 
conhecer 
>e já estabelecer um pré-conceito desses. 
> 
>>Em 8 May 2004, [EMAIL PROTECTED] escreveu: 
>> 
>>>a=15*t^2-15t-995 
>>>b=15*t^2-20t-992 
>>>c=15*t^2-18t-993 
>>> 
>>> --- Johann Peter Gustav Lejeune Dirichlet 
>>> escreveu: > E 
>>>so usar umas equaçoes de Pell e o problema 
 sai. 
 A verdadeira treta ai e achar Todas e somente 
 TODAS as soluçoes. Encara essa! 
 
 --- [EMAIL PROTECTED] escreveu: > Essa é 
 boa.Demonstrar que existem infinitos 
 > ternos (a,b,c),com a,b,c números 
 > naturais,que satisfazem a 
 > relação:2a^2+3b^2-5c^2=1997.Abraços vieira. 
 > 
 > 
 
>>>_ 
 > Voce quer um iGMail protegido contra vírus e 
 > spams? 
 > Clique aqui: 
 http://www.igmailseguro.ig.com.br 
 > Ofertas imperdíveis! Link: 
 > http://www.americanas.com.br/ig/ 
 > 
 > 
 
>>>= 
 > Instruções para entrar na lista, sair da 
 lista 
 > e usar a lista em 
 > 
 
>>>http://www.mat.puc-rio.br/~nicolau/olimp/obm-l.html 
 > 
 
>>>= 
 
 
 = 
 TRANSIRE SVVM PECTVS MVNDOQVE POTIRI 
 
 CONGREGATI EX TOTO ORBE MATHEMATICI OB SCRIPTA 
 INSIGNIA TRIBVERE 
 
 Fields Medal(John Charles Fields) 
 
 N.F.C. (Ne Fronti Crede) 
 
 
 
 
>>>__ 
 
 Yahoo! Messenger - Fale com seus amigos online. 
 Instale agora! 
 http://br.download.yahoo.com/messenger/ 
 
>>>= 
 Instruções para entrar na lista, sair da lista 
 e usar a lista em 
 
>>>http://www.mat.puc-rio.br/~nicolau/olimp/obm-l.html 
 
>>>= 
>>> 
>>>= 
>>>TRANSIRE SVVM PECTVS MVNDOQVE POTIRI 
>>> 
>>>CONGREGATI EX TOTO ORBE MATHEMATICI OB SCRIPTA INSIGNIA TRIBVERE 
>>> 
>>>Fields Medal(John Charles Fields) 
>>> 
>>>N.F.C. (Ne Fronti Crede) 
>>> 
>>>__ 
>>> 
>>>Yahoo! Messenger - Fale com seus amigos online. Instale agora! 
>>>http://br.download.yahoo.com/messenger/ 
>>>= 
>>>Instruções para entrar na lista, sair da lista e usar a lista em 
>>>http://www.mat.puc-rio.br/~nicolau/olimp/obm-l.html 
>>>= 
>>> 
>>>-- 
>> 
>>_ 
>>Voce quer um iGMail protegido contra vírus e spams? 
>>Clique aqui: http://www.igmailseguro.ig.com.br 
>> 
>>-- 
> 
>_ 
>Voce quer um iGMail protegido contra vírus e spams? 
>Clique aqui: http://www.igmailseguro.ig.com.br 
>Ofertas imperdíveis! Link: http://www.americanas.com.br/ig/ 
> 
>= 
>Instruções para entrar na lista, sair da lista e usar a lista em 
>http://www.mat.puc-rio.br/~nicolau/olimp/obm-l.html 
>= 
> 
>-- 

_
Voce quer um iGMail protegido contra vírus e spams? 
Clique aqui: http://www.igmailseguro.ig.com.br



Re: [obm-l] Questão de 2o. grau

2004-05-09 Por tôpico Claudio Buffara
Title: Re: [obm-l] Questão de 2o. grau



Uma variacao interessante dessa questao jah apareceu aqui na lista:

Ache todas as solucoes inteiras e positivas de:
1! + 2! + ... + n! = m^2
e prove que estas sao, de fato, as unicas.

[]s,
Claudio.

on 09.05.04 17:42, Fellipe Rossi at [EMAIL PROTECTED] wrote:

Como vocês demonstrariam, para 2o. grau, que 
 
para n>=1, n pertence a Z. apenas n=1 e n=3 são raízes da equação:
 
1!+2!+3!+...+n! = n^2 
 
Vocês aceitariam uma resolução que mostrasse, com exemplos (4!=24, 4^2=16 ; 5!=120, 5^2=25, e assim por diante...) que para n>=4. n! é maior que n^2 e que como o lado esquerdo da igualdade eh n!+valor positivo, ela vai ser sempre maior que o lado direito para n>=4, e substituindo n por 1, 2 e 3 chegamos q apenas 1 e 3 são raizes?
 
Essa qustão caiu, se não me engano, na prova específica da UFRJ 1992.
 
Abraços! 
Rossi







[obm-l] Demonstração

2004-05-09 Por tôpico Everton A. Ramos (www.bs2.com.br)
Olá

Qual a maneira correta e mais lógica de fazer uma demonstração:

Para a e b em R quaisquer prove que a(-b) = -(ab) = (-a)b




--
Everton Antonio Ramos (44) 8801-0186
[EMAIL PROTECTED]

Av. Dr. Luiz Teixeira Mendes, 638
Maringá - Paraná
(44) 3028-6300
--

- Original Message - 
From: "Augusto Cesar de Oliveira Morgado" <[EMAIL PROTECTED]>
To: <[EMAIL PROTECTED]>
Sent: Sunday, May 09, 2004 10:53 PM
Subject: Re: [obm-l] 8a cone sul


> 10a+b-a^2-b^2 = 25,25 -(a-5)^2 - (b-0,5)^2
> a = 5 e b=0 ou b=1
> ==
> Mensagem  enviada  pelo  CIP  WebMAIL  - Nova Geração - v. 2.1
> CentroIn Internet Provider  http://www.centroin.com.br
> Tel: (21) 2542-4849, (21) 2295-3331Fax: (21) 2295-2978
> Empresa 100% Brasileira - Desde 1992 prestando servicos online
>
>
> -- Original Message ---
> From: [EMAIL PROTECTED]
> To: [EMAIL PROTECTED]
> Sent: Sun, 9 May 2004 21:43:41 -0300
> Subject: [obm-l] 8a cone sul
>
> > >>Por favor alguém tem idea de como posso resolver esse problema. De
cada
> > número inteiro positivo n,n<=99,subtraímos a soma dos quadrados de
> > seus algarismos.Para que valores de n esta diferença é a maior
> > possível?
> >
> > _
> > Voce quer um iGMail protegido contra vírus e spams?
> > Clique aqui: http://www.igmailseguro.ig.com.br
> > Ofertas imperdíveis! Link: http://www.americanas.com.br/ig/
> >
> >
=
> > Instruções para entrar na lista, sair da lista e usar a lista em
> > http://www.mat.puc-rio.br/~nicolau/olimp/obm-l.html
> >
=
> --- End of Original Message ---
>
> =
> Instruções para entrar na lista, sair da lista e usar a lista em
> http://www.mat.puc-rio.br/~nicolau/olimp/obm-l.html
> =
>


=
Instruções para entrar na lista, sair da lista e usar a lista em
http://www.mat.puc-rio.br/~nicolau/olimp/obm-l.html
=


Re: [obm-l] olimpíada Russa

2004-05-09 Por tôpico Claudio Buffara
on 09.05.04 21:19, [EMAIL PROTECTED] at [EMAIL PROTECTED] wrote:

> os algarismos de um inteiro positivo A em sua representação decimal no
> sistema de numeração decimal cressem da esquerda para direita.Determine a
> soma dos algarismos do número 9.A
> 
A = a*10^n + b*10^(n-1) + c*10^(n-2) + ... + g*10 + h,
onde 1 <= a < b < c < ... < g < h <= 9  (*)

10A = a*10^(n+1) + b*10^n + c*10^(n-1) + ... + g*10^2 + h*10 + 0

Escreva 10A como:
10A = a*10^(n+1) + b*10^n + c*10^(n-1) + ... + g*10^2 + (h-1)*10 + 10
  A =  a*10^n + b*10^(n-1) + ... + f*10^2 + g*10 +  h

Subtraindo:
9A = a*10^(n-1) + (b-a)*10^n + (c-b)*10^(n-1) + ... + (h-1-g)*10 + (10-h)

Em virtude das desigualdades (*) acima, a soma dos algarismos de 9A serah:
a + (b-a) + (c-b) + ... + (g-f) + (h-1-g) + (10-h) = 10 - 1 = 9


[]s,
Claudio.


=
Instruções para entrar na lista, sair da lista e usar a lista em
http://www.mat.puc-rio.br/~nicolau/olimp/obm-l.html
=


Re: [obm-l] 8a cone sul

2004-05-09 Por tôpico Augusto Cesar de Oliveira Morgado
10a+b-a^2-b^2 = 25,25 -(a-5)^2 - (b-0,5)^2
a = 5 e b=0 ou b=1
==
Mensagem  enviada  pelo  CIP  WebMAIL  - Nova Geração - v. 2.1
CentroIn Internet Provider  http://www.centroin.com.br
Tel: (21) 2542-4849, (21) 2295-3331Fax: (21) 2295-2978
Empresa 100% Brasileira - Desde 1992 prestando servicos online


-- Original Message ---
From: [EMAIL PROTECTED]
To: [EMAIL PROTECTED]
Sent: Sun, 9 May 2004 21:43:41 -0300
Subject: [obm-l] 8a cone sul

> >>Por favor alguém tem idea de como posso resolver esse problema. De cada 
> número inteiro positivo n,n<=99,subtraímos a soma dos quadrados de 
> seus algarismos.Para que valores de n esta diferença é a maior 
> possível?
> 
> _
> Voce quer um iGMail protegido contra vírus e spams? 
> Clique aqui: http://www.igmailseguro.ig.com.br
> Ofertas imperdíveis! Link: http://www.americanas.com.br/ig/
> 
> =
> Instruções para entrar na lista, sair da lista e usar a lista em
> http://www.mat.puc-rio.br/~nicolau/olimp/obm-l.html
> =
--- End of Original Message ---

=
Instruções para entrar na lista, sair da lista e usar a lista em
http://www.mat.puc-rio.br/~nicolau/olimp/obm-l.html
=


Fw: [obm-l] 8a.cone sul

2004-05-09 Por tôpico Fellipe Rossi
Desculpem!! Faltou dizer que por congruência PT = TR


- Original Message -
From: "Fellipe Rossi" <[EMAIL PROTECTED]>
To: <[EMAIL PROTECTED]>
Sent: Sunday, May 09, 2004 9:50 PM
Subject: Re: [obm-l] 8a.cone sul


> Vamos la! :)
> Vejamos se ta certo
>
> Dificil explicar sem um desenho, aqueles q forem pacientes por favor lapis
e
> papel :)
>
> Assuma que o ponto onde a paralela ao diametro corta BR seja S
>
> Pegue o triângulo AOP, de ângulos P=90, O=b A=a,
> Devido à reta paralela, o triãngulo QPT tem os mesmos ângulos.
> Então fazendo a semelhança e chamando AO de 2r, usando o fato de QP ser
> metade de OP, vemos que QT = r
>
> Logo, QT é base média do triângulo AOH (metade da base e paralela a base),
> então sabemos que OT = OH.
>
> Agora tome os triângulos ABR e AOP que são semelhantes visto que ABR tem
um
> lado sobre o diametro então é retãngulo.
>
> OBSQ é um paralelogramo, temos que QS = 2r, então TS = r
>
> Assim, como OPT e TRH são semelhantes (do tipo LaL), temos que (ângulo
TRH)
> é reto, o que nos leva a concluir que B, R e H são colineares.
>
> Válido?
>
> Abraços do Rossi
>
>
> > >>- Original Message -
> > >>From:
> > >>To: ;
> > >>Sent: Sunday, May 09, 2004 4:11 PM
> > >>Subject: Re: [obm-l] 8a.cone sul
> > >>
> > >>> Em 8 May 2004, [EMAIL PROTECTED] escreveu:
> > >>>
> > >>> >Show de bola.Seja C uma circunferência de centro O, AB um diâmetro
> dela
> > >e
> > >>R
> > >>> >um ponto qualquer em C distinto de A e de B.Seja P a interseçâo da
> > >>> >perpendicular traçada por O a AR.Sobre a reta OP se marca o ponto
> Q,de
> > >>> >maneira que QP é a metade de PO e Q não pertence ao segmento OP.Por
Q
> > >>traçamos
> > >>> a
> > >>> >paralela a AB que corta a reta AR em T.Chamamos de H o ponto de
> > >>interseção
> > >>> >das retas AQ e OT.Provar que H,R e B são colineares.
> > >>> >
>
>


=
Instruções para entrar na lista, sair da lista e usar a lista em
http://www.mat.puc-rio.br/~nicolau/olimp/obm-l.html
=


Re: [obm-l] Demonstracoes no ensino medio

2004-05-09 Por tôpico Augusto Cesar de Oliveira Morgado
Ah, ah, ah!
Com esses livros didaticos e esses salarios de professores, a Matematica no 
ensino medio nao tem mais teoremas, tem, no maximo, observações.
O Provão mostrou que cerca de 20% dos licenciandos em Matemática (aí 
incluidos os formandos de universidades conceituadissimas) não sabem deduzir 
a fórmula que resolve equação do segundo grau.
Morgado

==
Mensagem  enviada  pelo  CIP  WebMAIL  - Nova Geração - v. 2.1
CentroIn Internet Provider  http://www.centroin.com.br
Tel: (21) 2542-4849, (21) 2295-3331Fax: (21) 2295-2978
Empresa 100% Brasileira - Desde 1992 prestando servicos online


-- Original Message ---
From: Claudio Buffara <[EMAIL PROTECTED]>
To: Lista OBM <[EMAIL PROTECTED]>
Sent: Sun, 09 May 2004 21:43:51 -0300
Subject: [obm-l] Demonstracoes no ensino medio

> Esta eh uma pergunta para quem estah cursando ou lecionando 
> matematica no ensino medio "normal" (ou seja, excluindo cursos de 
> preparacao para olimpiadas ou vestibulares muito puxados como IME e 
> ITA):
> 
> Em provas e exercicios propostos exige-se que os alunos apresentem
> demonstracoes de resultados matematicos ou apenas pede-se que eles calculem
> coisas?
> 
> []s,
> Claudio.
> 
> =
> Instruções para entrar na lista, sair da lista e usar a lista em
> http://www.mat.puc-rio.br/~nicolau/olimp/obm-l.html
> =
--- End of Original Message ---

=
Instruções para entrar na lista, sair da lista e usar a lista em
http://www.mat.puc-rio.br/~nicolau/olimp/obm-l.html
=


Re: [obm-l] 8a.cone sul

2004-05-09 Por tôpico Fellipe Rossi
Vamos la! :)
Vejamos se ta certo

Dificil explicar sem um desenho, aqueles q forem pacientes por favor lapis e
papel :)

Assuma que o ponto onde a paralela ao diametro corta BR seja S

Pegue o triângulo AOP, de ângulos P=90, O=b A=a,
Devido à reta paralela, o triãngulo QPT tem os mesmos ângulos.
Então fazendo a semelhança e chamando AO de 2r, usando o fato de QP ser
metade de OP, vemos que QT = r

Logo, QT é base média do triângulo AOH (metade da base e paralela a base),
então sabemos que OT = OH.

Agora tome os triângulos ABR e AOP que são semelhantes visto que ABR tem um
lado sobre o diametro então é retãngulo.

OBSQ é um paralelogramo, temos que QS = 2r, então TS = r

Assim, como OPT e TRH são semelhantes (do tipo LaL), temos que (ângulo TRH)
é reto, o que nos leva a concluir que B, R e H são colineares.

Válido?

Abraços do Rossi


> >>- Original Message -
> >>From:
> >>To: ;
> >>Sent: Sunday, May 09, 2004 4:11 PM
> >>Subject: Re: [obm-l] 8a.cone sul
> >>
> >>> Em 8 May 2004, [EMAIL PROTECTED] escreveu:
> >>>
> >>> >Show de bola.Seja C uma circunferência de centro O, AB um diâmetro
dela
> >e
> >>R
> >>> >um ponto qualquer em C distinto de A e de B.Seja P a interseçâo da
> >>> >perpendicular traçada por O a AR.Sobre a reta OP se marca o ponto
Q,de
> >>> >maneira que QP é a metade de PO e Q não pertence ao segmento OP.Por Q
> >>traçamos
> >>> a
> >>> >paralela a AB que corta a reta AR em T.Chamamos de H o ponto de
> >>interseção
> >>> >das retas AQ e OT.Provar que H,R e B são colineares.
> >>> >



=
Instruções para entrar na lista, sair da lista e usar a lista em
http://www.mat.puc-rio.br/~nicolau/olimp/obm-l.html
=


[obm-l] 8a cone sul

2004-05-09 Por tôpico vieirausp
>>Por favor alguém tem idea de como posso resolver esse problema. De cada 
número inteiro positivo n,n<=99,subtraímos a soma dos quadrados de seus 
algarismos.Para que valores de n esta diferença é a maior possível? 

_
Voce quer um iGMail protegido contra vírus e spams? 
Clique aqui: http://www.igmailseguro.ig.com.br
Ofertas imperdíveis! Link: http://www.americanas.com.br/ig/

=
Instruções para entrar na lista, sair da lista e usar a lista em
http://www.mat.puc-rio.br/~nicolau/olimp/obm-l.html
=


[obm-l] Demonstracoes no ensino medio

2004-05-09 Por tôpico Claudio Buffara
Esta eh uma pergunta para quem estah cursando ou lecionando matematica no
ensino medio "normal" (ou seja, excluindo cursos de preparacao para
olimpiadas ou vestibulares muito puxados como IME e ITA):

Em provas e exercicios propostos exige-se que os alunos apresentem
demonstracoes de resultados matematicos ou apenas pede-se que eles calculem
coisas?

[]s,
Claudio.

=
Instruções para entrar na lista, sair da lista e usar a lista em
http://www.mat.puc-rio.br/~nicolau/olimp/obm-l.html
=


[obm-l] Olimpíada Chinesa

2004-05-09 Por tôpico vieirausp
Seja PQRS um quadrilátero inscrito num círculo e cuja medida do ângulo PSR 
seja igual a 90.Se H e k são os pés das perpendiculares baixadas de Q sobre 
PR e PS respectivamente (convenientemente prologados se necessário).Mostre 
que HK divide QS ao meio. 

_
Voce quer um iGMail protegido contra vírus e spams? 
Clique aqui: http://www.igmailseguro.ig.com.br
Ofertas imperdíveis! Link: http://www.americanas.com.br/ig/

=
Instruções para entrar na lista, sair da lista e usar a lista em
http://www.mat.puc-rio.br/~nicolau/olimp/obm-l.html
=


Re: [obm-l] Re: [obm-l] Re: [obm-l] Questão de 2o. grau

2004-05-09 Por tôpico Augusto Cesar de Oliveira Morgado



A melhor maneira de transformar uma pessoa (honesta, por suposto) em defensora incondicional das provas de multipla escolha é faze-la acompanhar de perto a correçao de provas discursivas de um vestibular. O criterio de correçao é nenhum!
Morgado

== 
Mensagem  enviada  pelo  CIP  WebMAIL  - Nova Geração - v. 2.1 
CentroIn Internet Provider          http://www.centroin.com.br 
Tel: (21) 2542-4849, (21) 2295-3331        Fax: (21) 2295-2978 
Empresa 100% Brasileira - Desde 1992 prestando servicos online 


-- Original Message ---
From: "Fellipe Rossi" <[EMAIL PROTECTED]> 
 O que eu queria saber é se uma Banca de vestibular aceita esses tipos de resolução. 





Re: [obm-l] Dominos e Fibonacci

2004-05-09 Por tôpico Claudio Buffara
"Serve pra que?" eh uma pergunta complicada de se responder quando diz
respeito a algum conceito matematico, ateh porque a maioria dos matematicos
a considera uma pergunta sem importancia. De qualquer forma, talvez voce
ache alguma resposta aqui:
http://www.mcs.surrey.ac.uk/Personal/R.Knott/Fibonacci/fibnat.html#rabeecow

[]s,
Claudio.

on 09.05.04 20:33, Fabiano Sant'Ana at [EMAIL PROTECTED] wrote:

> Fibonacci serve pra que?
> 
> fabiano
> "que só usou fibonacci numa prova de Algoritmos de programação (Era um
> exercicio da prova)"
> - Original Message -
> From: "Claudio Buffara" <[EMAIL PROTECTED]>
> To: "Lista OBM" <[EMAIL PROTECTED]>
> Sent: Sunday, May 09, 2004 7:49 PM
> Subject: [obm-l] Dominos e Fibonacci
> 
> 


=
Instruções para entrar na lista, sair da lista e usar a lista em
http://www.mat.puc-rio.br/~nicolau/olimp/obm-l.html
=


[obm-l] olimpíada Russa

2004-05-09 Por tôpico vieirausp
os algarismos de um inteiro positivo A em sua representação decimal no 
sistema de numeração decimal cressem da esquerda para direita.Determine a 
soma dos algarismos do número 9.A 

_
Voce quer um iGMail protegido contra vírus e spams?
Clique aqui: http://www.igmailseguro.ig.com.br
Ofertas imperdíveis! Link: http://www.americanas.com.br/ig/

=
Instruções para entrar na lista, sair da lista e usar a lista em
http://www.mat.puc-rio.br/~nicolau/olimp/obm-l.html
=


[obm-l] Re: [obm-l] Re: [obm-l] Questão de 2o. grau

2004-05-09 Por tôpico Fellipe Rossi



Sim, mas essas demonstrações exigem uma experiência 
raramente encontrada em alunos de 2o. grau.
 
Outra questão do tipo seria mostrar que pra n>1, 
n pertence a N, que (n^2)! > (n!)^2
 
No caso eu faria:
 
(n^2)! = 1*2*3*...*n^2 = 
1*2*3...*n*(n+1)*(n+2)*...*n^2
(n!)^2= 1*1*2*2*3*3*4*4*...*n*n = 
1*2*3...*n*1*2*3...*n 
 
como (n+1)(n+2)...n^2 > 
1*2*3...*n    pois n+1>1, n+2>2,..., n^2>n 
provamos
 
Porém é mais um tipo de resolução que raramente 
entra na cabeça de vestibulandos.
 
Oq é mais normal fazer seria:
(2^2)! = 24
(2!)^2 = 4
 
(3^2)! = 9!
(3!)^2 = 36
 
para valores maiores de n maior será a 
diferença...
 
O que eu queria saber é se uma Banca de vestibular 
aceita esses tipos de resolução.
 
Grato pela atenção!
Rossi

  - Original Message - 
  From: 
  Marcio Afonso A. Cohen 
  To: [EMAIL PROTECTED] 
  Sent: Sunday, May 09, 2004 6:14 PM
  Subject: [obm-l] Re: [obm-l] Questão de 
  2o. grau
  
      Em parte. Tudo que voce diz eh 
  verdade, mas eu exigiria uma explicacao um pouquinho melhor de pq n! eh maior 
  que n^2. Mas a ideia eh otima e funciona. Eu acho q faria algo como: p/ 
  n>3, 1! + 2! + ... + n! >= n! + (n-1)!+1 > n(n-1) + (n-1) + 1 = 
  n^2.
      Uma outra opcao eh olhar mod 
  10.
  
- Original Message - 
From: 
Fellipe Rossi 
To: [EMAIL PROTECTED] 
Sent: Sunday, May 09, 2004 5:42 
PM
Subject: [obm-l] Questão de 2o. 
grau

Como vocês demonstrariam, para 2o. grau, que 

 
para n>=1, n pertence a Z. apenas n=1 e n=3 
são raízes da equação:
 
1!+2!+3!+...+n! = n^2 
 
Vocês aceitariam uma resolução que mostrasse, 
com exemplos (4!=24, 4^2=16 ; 5!=120, 5^2=25, e assim por diante...) que 
para n>=4. n! é maior que n^2 e que como o lado esquerdo da igualdade eh 
n!+valor positivo, ela vai ser sempre maior que o lado direito para n>=4, 
e substituindo n por 1, 2 e 3 chegamos q apenas 1 e 3 são 
raizes?
 
Essa qustão caiu, se não me engano, na prova 
específica da UFRJ 1992.
 
Abraços! 
Rossi


Re: [obm-l] 8a.cone sul

2004-05-09 Por tôpico vieirausp
Em 9 May 2004, [EMAIL PROTECTED] escreveu: 

>Cara, não leva a mal. 
>Você continua não colaborando com coisa alguma. Ou você faz observações que 
>não acrescentam nada, ou faz este tipo de intervenção mágica. Isto é pouco 
>instrutivo. O que são as equações de Pell? Por que você não explica para 
>todos? Eu não sei o que são estas equações e resolvi. Acho que você é o 
>maior enrolão. Pare de embromar e faça algo de construtivo como o Buffara, 
>Steiner, Morgado, Nicolau, Santa Rita, Cyberhelp, etc. 
>Tenho pena de quem vir a ser seu aluno. 
>Passe bem. 

>Não é nada disso, sou muito esforçado e estou tentando aprender, se voçê 
não pode me ajudar não me desanime.Pena eu tenho de voçê por não me conhecer 
e já estabelecer um pré-conceito desses. 

>Em 8 May 2004, [EMAIL PROTECTED] escreveu: 
> 
>>a=15*t^2-15t-995 
>>b=15*t^2-20t-992 
>>c=15*t^2-18t-993 
>> 
>> --- Johann Peter Gustav Lejeune Dirichlet 
>> escreveu: > E 
>>so usar umas equaçoes de Pell e o problema 
>>> sai. 
>>> A verdadeira treta ai e achar Todas e somente 
>>> TODAS as soluçoes. Encara essa! 
>>> 
>>> --- [EMAIL PROTECTED] escreveu: > Essa é 
>>> boa.Demonstrar que existem infinitos 
>>> > ternos (a,b,c),com a,b,c números 
>>> > naturais,que satisfazem a 
>>> > relação:2a^2+3b^2-5c^2=1997.Abraços vieira. 
>>> > 
>>> > 
>>> 
>>_ 
>>> > Voce quer um iGMail protegido contra vírus e 
>>> > spams? 
>>> > Clique aqui: 
>>> http://www.igmailseguro.ig.com.br 
>>> > Ofertas imperdíveis! Link: 
>>> > http://www.americanas.com.br/ig/ 
>>> > 
>>> > 
>>> 
>>= 
>>> > Instruções para entrar na lista, sair da 
>>> lista 
>>> > e usar a lista em 
>>> > 
>>> 
>>http://www.mat.puc-rio.br/~nicolau/olimp/obm-l.html 
>>> > 
>>> 
>>= 
>>> 
>>> 
>>> = 
>>> TRANSIRE SVVM PECTVS MVNDOQVE POTIRI 
>>> 
>>> CONGREGATI EX TOTO ORBE MATHEMATICI OB SCRIPTA 
>>> INSIGNIA TRIBVERE 
>>> 
>>> Fields Medal(John Charles Fields) 
>>> 
>>> N.F.C. (Ne Fronti Crede) 
>>> 
>>> 
>>> 
>>> 
>>__ 
>>> 
>>> Yahoo! Messenger - Fale com seus amigos online. 
>>> Instale agora! 
>>> http://br.download.yahoo.com/messenger/ 
>>> 
>>= 
>>> Instruções para entrar na lista, sair da lista 
>>> e usar a lista em 
>>> 
>>http://www.mat.puc-rio.br/~nicolau/olimp/obm-l.html 
>>> 
>>= 
>> 
>>= 
>>TRANSIRE SVVM PECTVS MVNDOQVE POTIRI 
>> 
>>CONGREGATI EX TOTO ORBE MATHEMATICI OB SCRIPTA INSIGNIA TRIBVERE 
>> 
>>Fields Medal(John Charles Fields) 
>> 
>>N.F.C. (Ne Fronti Crede) 
>> 
>>__ 
>> 
>>Yahoo! Messenger - Fale com seus amigos online. Instale agora! 
>>http://br.download.yahoo.com/messenger/ 
>>= 
>>Instruções para entrar na lista, sair da lista e usar a lista em 
>>http://www.mat.puc-rio.br/~nicolau/olimp/obm-l.html 
>>= 
>> 
>>-- 
> 
>_ 
>Voce quer um iGMail protegido contra vírus e spams? 
>Clique aqui: http://www.igmailseguro.ig.com.br 
> 
>-- 

_
Voce quer um iGMail protegido contra vírus e spams? 
Clique aqui: http://www.igmailseguro.ig.com.br
Ofertas imperdíveis! Link: http://www.americanas.com.br/ig/

=
Instruções para entrar na lista, sair da lista e usar a lista em
http://www.mat.puc-rio.br/~nicolau/olimp/obm-l.html
=


Re: [obm-l] Dominos e Fibonacci

2004-05-09 Por tôpico Fabiano Sant'Ana
Fibonacci serve pra que?

fabiano
"que só usou fibonacci numa prova de Algoritmos de programação (Era um
exercicio da prova)"
- Original Message -
From: "Claudio Buffara" <[EMAIL PROTECTED]>
To: "Lista OBM" <[EMAIL PROTECTED]>
Sent: Sunday, May 09, 2004 7:49 PM
Subject: [obm-l] Dominos e Fibonacci


> De quantas maneiras podemos cobrir um tabuleiro 2xn com dominos?
> Suponha que os dominos sao simetricos (ou seja, ambos os quadrados tem o
> mesmo numero de bolinhas).
>
> =
> Instruções para entrar na lista, sair da lista e usar a lista em
> http://www.mat.puc-rio.br/~nicolau/olimp/obm-l.html
> =
>


---
Outgoing mail is certified Virus Free.
Checked by AVG anti-virus system (http://www.grisoft.com).
Version: 6.0.676 / Virus Database: 438 - Release Date: 03/05/04

=
Instruções para entrar na lista, sair da lista e usar a lista em
http://www.mat.puc-rio.br/~nicolau/olimp/obm-l.html
=


[obm-l] Dominos e Fibonacci

2004-05-09 Por tôpico Claudio Buffara
De quantas maneiras podemos cobrir um tabuleiro 2xn com dominos?
Suponha que os dominos sao simetricos (ou seja, ambos os quadrados tem o
mesmo numero de bolinhas).

=
Instruções para entrar na lista, sair da lista e usar a lista em
http://www.mat.puc-rio.br/~nicolau/olimp/obm-l.html
=


Re: [obm-l] 8a.cone sul

2004-05-09 Por tôpico Fabio Henrique
Cara, não leva a mal. 
Você continua não colaborando com coisa alguma. Ou você faz observações que 
não acrescentam nada, ou faz este tipo de intervenção mágica. Isto é pouco 
instrutivo. O que são as equações de Pell? Por que você não explica para 
todos? Eu não sei o que são estas equações e resolvi. Acho que você é o 
maior enrolão. Pare de embromar e faça algo de construtivo como o Buffara, 
Steiner, Morgado, Nicolau, Santa Rita, Cyberhelp, etc. 
Tenho pena de quem vir a ser seu aluno. 
Passe bem. 



Em 8 May 2004, [EMAIL PROTECTED] escreveu: 

>a=15*t^2-15t-995 
>b=15*t^2-20t-992 
>c=15*t^2-18t-993 
> 
> --- Johann Peter Gustav Lejeune Dirichlet 
> escreveu: > E 
>so usar umas equaçoes de Pell e o problema 
>> sai. 
>> A verdadeira treta ai e achar Todas e somente 
>> TODAS as soluçoes. Encara essa! 
>> 
>> --- [EMAIL PROTECTED] escreveu: > Essa é 
>> boa.Demonstrar que existem infinitos 
>> > ternos (a,b,c),com a,b,c números 
>> > naturais,que satisfazem a 
>> > relação:2a^2+3b^2-5c^2=1997.Abraços vieira. 
>> > 
>> > 
>> 
>_ 
>> > Voce quer um iGMail protegido contra vírus e 
>> > spams? 
>> > Clique aqui: 
>> http://www.igmailseguro.ig.com.br 
>> > Ofertas imperdíveis! Link: 
>> > http://www.americanas.com.br/ig/ 
>> > 
>> > 
>> 
>= 
>> > Instruções para entrar na lista, sair da 
>> lista 
>> > e usar a lista em 
>> > 
>> 
>http://www.mat.puc-rio.br/~nicolau/olimp/obm-l.html 
>> > 
>> 
>= 
>> 
>> 
>> = 
>> TRANSIRE SVVM PECTVS MVNDOQVE POTIRI 
>> 
>> CONGREGATI EX TOTO ORBE MATHEMATICI OB SCRIPTA 
>> INSIGNIA TRIBVERE 
>> 
>> Fields Medal(John Charles Fields) 
>> 
>> N.F.C. (Ne Fronti Crede) 
>> 
>> 
>> 
>> 
>__ 
>> 
>> Yahoo! Messenger - Fale com seus amigos online. 
>> Instale agora! 
>> http://br.download.yahoo.com/messenger/ 
>> 
>= 
>> Instruções para entrar na lista, sair da lista 
>> e usar a lista em 
>> 
>http://www.mat.puc-rio.br/~nicolau/olimp/obm-l.html 
>> 
>= 
> 
>= 
>TRANSIRE SVVM PECTVS MVNDOQVE POTIRI 
> 
>CONGREGATI EX TOTO ORBE MATHEMATICI OB SCRIPTA INSIGNIA TRIBVERE 
> 
>Fields Medal(John Charles Fields) 
> 
>N.F.C. (Ne Fronti Crede) 
> 
>__ 
> 
>Yahoo! Messenger - Fale com seus amigos online. Instale agora! 
>http://br.download.yahoo.com/messenger/ 
>= 
>Instruções para entrar na lista, sair da lista e usar a lista em 
>http://www.mat.puc-rio.br/~nicolau/olimp/obm-l.html 
>= 
> 
>-- 

_
Voce quer um iGMail protegido contra vírus e spams? 
Clique aqui: http://www.igmailseguro.ig.com.br



Re: [obm-l] Re:_[obm-l]_Questão_de_2o._g rau

2004-05-09 Por tôpico vieirausp
Em 9 May 2004, [EMAIL PROTECTED] escreveu: 

>>Show de bola.Seja C uma circunferência de centro O, AB um diâmetro dela 
e 
>R 
>> >um ponto qualquer em C distinto de A e de B.Seja P a interseçâo da 
>> >perpendicular traçada por O a AR.Sobre a reta OP se marca o ponto Q,de 
>> >maneira que QP é a metade de PO e Q não pertence ao segmento OP.Por Q 
>traçamos 
>> a 
>> >paralela a AB que corta a reta AR em T.Chamamos de H o ponto de 
>interseção 
>> >das retas AQ e OT.Provar que H,R e B são colineares. 
>> > abraços vieira. 

>-- 

_
Voce quer um iGMail protegido contra vírus e spams?
Clique aqui: http://www.igmailseguro.ig.com.br
Ofertas imperdíveis! Link: http://www.americanas.com.br/ig/

=
Instruções para entrar na lista, sair da lista e usar a lista em
http://www.mat.puc-rio.br/~nicolau/olimp/obm-l.html
=


Re: [obm-l] 8a.cone sul

2004-05-09 Por tôpico vieirausp
Em 9 May 2004, [EMAIL PROTECTED] escreveu: 

>Em 9 May 2004, [EMAIL PROTECTED] escreveu: 
> 
>>ta mto confuso o enunciado 
>>tem certeza q transcreveu corretamente? 
>>Sim, o enunciado está correto tal qual se encontra no arquivo de provas da 
>obm(8a cone sul) 
>>- Original Message - 
>>From: 
>>To: ; 
>>Sent: Sunday, May 09, 2004 4:11 PM 
>>Subject: Re: [obm-l] 8a.cone sul 
>> 
>>> Em 8 May 2004, [EMAIL PROTECTED] escreveu: 
>>> 
>>> >Show de bola.Seja C uma circunferência de centro O, AB um diâmetro dela 
>e 
>>R 
>>> >um ponto qualquer em C distinto de A e de B.Seja P a interseçâo da 
>>> >perpendicular traçada por O a AR.Sobre a reta OP se marca o ponto Q,de 
>>> >maneira que QP é a metade de PO e Q não pertence ao segmento OP.Por Q 
>>traçamos 
>>> a 
>>> >paralela a AB que corta a reta AR em T.Chamamos de H o ponto de 
>>interseção 
>>> >das retas AQ e OT.Provar que H,R e B são colineares. 
>>> > 
>>> >_ 
>>> >Voce quer um iGMail protegido contra vírus e spams? 
>>> >Clique aqui: http://www.igmailseguro.ig.com.br 
>>> >Ofertas imperdíveis! Link: http://www.americanas.com.br/ig/ 
>>> > 
>>> 
>>= 
>>> >Instruções para entrar na lista, sair da lista e usar a lista em 
>>> >http://www.mat.puc-rio.br/~nicolau/olimp/obm-l.html 
>>> 
>>= 
>>> > 
>>> >-- 
>>> 
>>> _ 
>>> Voce quer um iGMail protegido contra vírus e spams? 
>>> Clique aqui: http://www.igmailseguro.ig.com.br 
>>> Ofertas imperdíveis! Link: http://www.americanas.com.br/ig/ 
>>> 
>>> 
= 
>>> Instruções para entrar na lista, sair da lista e usar a lista em 
>>> http://www.mat.puc-rio.br/~nicolau/olimp/obm-l.html 
>>> 
= 
>>> 
>>> 
>> 
>>= 
>>Instruções para entrar na lista, sair da lista e usar a lista em 
>>http://www.mat.puc-rio.br/~nicolau/olimp/obm-l.html 
>>= 
>> 
>>-- 
> 
>_ 
>Voce quer um iGMail protegido contra vírus e spams? 
>Clique aqui: http://www.igmailseguro.ig.com.br 
>Ofertas imperdíveis! Link: http://www.americanas.com.br/ig/ 
> 
>= 
>Instruções para entrar na lista, sair da lista e usar a lista em 
>http://www.mat.puc-rio.br/~nicolau/olimp/obm-l.html 
>= 
> 
>-- 

_
Voce quer um iGMail protegido contra vírus e spams? 
Clique aqui: http://www.igmailseguro.ig.com.br
Ofertas imperdíveis! Link: http://www.americanas.com.br/ig/

=
Instruções para entrar na lista, sair da lista e usar a lista em
http://www.mat.puc-rio.br/~nicolau/olimp/obm-l.html
=


Re: [obm-l] Xadrez

2004-05-09 Por tôpico vieirausp
Em 9 May 2004, [EMAIL PROTECTED] escreveu:E ai 
Fábio beleza gostaria que me desse uma pista de como resolver este 
problema,já sei que é com equações de Pell mas como chegar a estas equações? 
> Demonstrar que existem infinitos 
> > ternos (a,b,c),com a,b,c números 
> > naturais,que satisfazem a 
> > relação:2a^2+3b^2-5c^2=1997.Abraços vieira. 
> > 

>> 

_
Voce quer um iGMail protegido contra vírus e spams?
Clique aqui: http://www.igmailseguro.ig.com.br
Ofertas imperdíveis! Link: http://www.americanas.com.br/ig/

=
Instruções para entrar na lista, sair da lista e usar a lista em
http://www.mat.puc-rio.br/~nicolau/olimp/obm-l.html
=


Re: [obm-l] Xadrez

2004-05-09 Por tôpico Fabio Dias Moreira

Welma Pereira said:
> Ok muito legal sua idéia Fábio, mas onde vao estar localizadas as
> probabilidades dos 4 cantos e das casas centrais nesta matriz 10X10 que
> vc  sugeriu?
> [...]

Você vai definir uma matriz P, 10x10, de tal forma que P_ij seja a
probabilidade de ir de uma casa com o número j escrito para uma casa onde
o número i esteja escrito. Encontre os autovetores dessa matriz -- só um
deles terá autovalor associado um, e é essa a sua solução. A probabilidade
do rei estar em uma casa com o número i é a i-ésima coordenada desse
autovetor.

Pelas minhas contas, se A = 3/420, B = 5/420, C = 8/420, então a
distribuição de probabilidades no tabuleiro é a seguinte:

ABBA
BCCB
BCCB
BCCB
BCCB
BCCB
BCCB
ABBA

Lembre-se que a i-ésima coordenada diz a probabilidade do rei estar em uma
casa com o número i, logo ela tem que ser dividida pelo número de casas
identificadas com o número i se você quer saber a probabilidade de cada
casa individual.

[]s,

-- 
Fábio "ctg \pi" Dias Moreira


=
Instruções para entrar na lista, sair da lista e usar a lista em
http://www.mat.puc-rio.br/~nicolau/olimp/obm-l.html
=


[obm-l] Re: [obm-l] Questão de 2o. grau

2004-05-09 Por tôpico Marcio Afonso A. Cohen



    Em parte. Tudo que voce diz eh 
verdade, mas eu exigiria uma explicacao um pouquinho melhor de pq n! eh maior 
que n^2. Mas a ideia eh otima e funciona. Eu acho q faria algo como: p/ 
n>3, 1! + 2! + ... + n! >= n! + (n-1)!+1 > n(n-1) + (n-1) + 1 = 
n^2.
    Uma outra opcao eh olhar mod 
10.

  - Original Message - 
  From: 
  Fellipe Rossi 
  To: [EMAIL PROTECTED] 
  Sent: Sunday, May 09, 2004 5:42 PM
  Subject: [obm-l] Questão de 2o. 
grau
  
  Como vocês demonstrariam, para 2o. grau, que 
  
   
  para n>=1, n pertence a Z. apenas n=1 e n=3 
  são raízes da equação:
   
  1!+2!+3!+...+n! = n^2 
   
  Vocês aceitariam uma resolução que mostrasse, com 
  exemplos (4!=24, 4^2=16 ; 5!=120, 5^2=25, e assim por diante...) que para 
  n>=4. n! é maior que n^2 e que como o lado esquerdo da igualdade eh 
  n!+valor positivo, ela vai ser sempre maior que o lado direito para n>=4, e 
  substituindo n por 1, 2 e 3 chegamos q apenas 1 e 3 são raizes?
   
  Essa qustão caiu, se não me engano, na prova 
  específica da UFRJ 1992.
   
  Abraços! 
  Rossi


Re: [obm-l] Xadrez

2004-05-09 Por tôpico Welma Pereira
Ok muito legal sua idéia Fábio, mas onde vao estar localizadas as 
probabilidades dos 4 cantos e das casas centrais nesta matriz 10X10 que vc 
sugeriu?


From: "Fabio Dias Moreira" <[EMAIL PROTECTED]>
Reply-To: [EMAIL PROTECTED]
To: <[EMAIL PROTECTED]>
Subject: Re: [obm-l] Xadrez
Date: Sat, 8 May 2004 19:51:16 -0300 (BRT)
Claudio Buffara said:
> on 07.05.04 19:16, Welma Pereira at [EMAIL PROTECTED] wrote:
>
>> Alguém poderia me ajudar com um problema sobre xadrez envolvendo
>> cadeia de Markov?
>>
>> O problema é o seguinte
>>
>> Um rei se mexe em um tabuleiro de xadrez escolhendo com a mesma
>> probabilidade um dos lances possiveis.Qual a probabilidade que após um
>> número grande de lances o rei se encontre em uma das 4 casas
>> centrais(d4,e4,d5,e5)? e  Qual a probabilidade que se encontre em um
>> dos 4 cantos(a1,h1,a8,h8)?
>>
> Forme a matriz de transicao P desse sistema (uma matriz 64x64), calcule
> P^n e veja o que acontece quando n -> infinito.
>
> Por exemplo, numere os quadrados do tabuleiro da seguinte forma:
> 01  02  03  ...  07  08
> 09  10  11  ...  15  16
> ..  ..  ..  ...  ..  ..
> 57  58  59  ...  63  64
>
> Em seguida, calcule p(i,j) = probabilidade do rei ir do quadrado i ao
> quadrado j.
>
> Por exemplo:
> p(k,k) = 0, para todo k; p(1,2) = 1/3; p(3,10) = 1/5; p(10,11) = 1/8;
> p(10,12) = 0.
>
> Eh meio sacal de fazer no braco, mas com um computador eh bem facil
> preencher a matriz P e calcular P^n para n grande.
> [...]
Na realidade, dá para fazer algo mais esperto: o tabuleiro de xadrez
possui dois eixos de simetria e um centro de simetria, logo conjuntos de 4
(ou até de 8) casas têm associações naturais, logo só é necessária uma
matriz 10x10:
01233210
14566541
25788752
36899863
36899863
25788752
14566541
01233210
Além disso, se lim A^n quando n->inf vale B, então AB = B, logo B é uma
matriz cujas colunas são todas autovetores de A, logo é possível obter uma
resposta exata para o valor dessas probabilidades.
[]s,
--
Fábio "ctg \pi" Dias Moreira
=
Instruções para entrar na lista, sair da lista e usar a lista em
http://www.mat.puc-rio.br/~nicolau/olimp/obm-l.html
=
_
MSN Hotmail, o maior webmail do Brasil.  http://www.hotmail.com
=
Instruções para entrar na lista, sair da lista e usar a lista em
http://www.mat.puc-rio.br/~nicolau/olimp/obm-l.html
=


[obm-l] Questão de 2o. grau

2004-05-09 Por tôpico Fellipe Rossi



Como vocês demonstrariam, para 2o. grau, que 

 
para n>=1, n pertence a Z. apenas n=1 e n=3 são 
raízes da equação:
 
1!+2!+3!+...+n! = n^2 
 
Vocês aceitariam uma resolução que mostrasse, com 
exemplos (4!=24, 4^2=16 ; 5!=120, 5^2=25, e assim por diante...) que para 
n>=4. n! é maior que n^2 e que como o lado esquerdo da igualdade eh n!+valor 
positivo, ela vai ser sempre maior que o lado direito para n>=4, e 
substituindo n por 1, 2 e 3 chegamos q apenas 1 e 3 são raizes?
 
Essa qustão caiu, se não me engano, na prova 
específica da UFRJ 1992.
 
Abraços! 
Rossi


[obm-l] RE: [obm-l] Princípio de Dirichlet - variacao

2004-05-09 Por tôpico Frederico Reis Marques de Brito
Convém então esclarecer novamente uma confusão. Embora não haja muito 
consenso a respeito das nomenclaturas entre círculo e circunferência, é mais 
comum referir-se a região delimitada por uma circunferência como disco, até 
porque essa nomenclatura não dá margem a ambiguidade. O problema que propus 
não foi o que o Ricardo resolveu, mas basicamente o que o Cláudio ora 
propõe.  Assim sendo, reformulo o enunciado , deixando-o mais exato e 
formal:

Considere o conjunto S dos pontos do R^2 que distam, na métrica euclidiana, 
1 unidade da origem do R^2. Se a cada ponto de S  associarmos um elemento do 
conjunto  T={A,B} então existirão sempre três pontos de S equidistantes ( na 
métrica euclidiana ) associados a um mesmo elemento de T.

Abraços a todos,
FRED.

From: Claudio Buffara <[EMAIL PROTECTED]>
Reply-To: [EMAIL PROTECTED]
To: <[EMAIL PROTECTED]>
Subject: [obm-l] Princípio de Dirichlet - variacao
Date: Sun, 09 May 2004 15:53:12 -0300
Frederico Reis Marques de Brito wrote:
>
> Se pintarmos cada ponto de um círculo com [uma dentre] duas cores,
> de forma aleatória, então existirão
> três pontos equidistantes pintados com a mesma cor.
>
E se ao inves de circulo (ou seja, disco) o enunciado falasse em
circunferencia (de modo que nao pudessemos usar o centro)?
Ainda teriamos um triangulo equilatero com os 3 vertices da mesma cor?
[]s,
Claudio.
=
Instruções para entrar na lista, sair da lista e usar a lista em
http://www.mat.puc-rio.br/~nicolau/olimp/obm-l.html
=
_
MSN Hotmail, o maior webmail do Brasil.  http://www.hotmail.com
=
Instruções para entrar na lista, sair da lista e usar a lista em
http://www.mat.puc-rio.br/~nicolau/olimp/obm-l.html
=


Re: [obm-l] Probabilidade e quadradinhos

2004-05-09 Por tôpico Rafael
Cláudio,

Muito obrigado pela solução. Creio, assim, que a minha também esteja
correta. Tomando coragem para as contas:

P = [4*2 + 4(n-2)*3 + (n^2 - 4 - 4(n-2))*4]/[n^2(n^2 - 1)] =
= (8 + 12n - 24 + 4n^2 - 16 - 16n + 32)/[n^2(n^2 - 1)] =
= 4n(n - 1)/[n^2(n^2 - 1)] = 4/[n(n+1)]

...


A sua idéia de utilizar a escolha de um dominó foi ótima. Eu não estou
certo, mas esse resultado não tem a ver com os números de Fibonacci? É fácil
demonstrá-lo?


Um abraço e obrigado de novo,

Rafael de A. Sampaio




- Original Message -
From: "Claudio Buffara" <[EMAIL PROTECTED]>
To: <[EMAIL PROTECTED]>
Sent: Sunday, May 09, 2004 10:52 AM
Subject: Re: [obm-l] Probabilidade e quadradinhos


Eu acho que eh assim:

Numero de casos possiveis =
Numero de maneiras de se escolher 2 quadrados =
Binom(n^2,2) = n^2*(n+1)*(n-1)/2

Numero de casos favoraveis =
Numero de maneiras de se escolher dois quadrados com um lado em comum =
Numero de maneiras de se escolher um "domino" =
2*n*(n-1)

Probabilidade = 2*n*(n-1)/((n^2*(n+1)*(n-1)/2) = 4/(n*(n+1))


[]s,
Claudio.


=
Instruções para entrar na lista, sair da lista e usar a lista em
http://www.mat.puc-rio.br/~nicolau/olimp/obm-l.html
=


[obm-l] Re: [obm-l] Re: [obm-l] Re: [obm-l] Fatoração ( IMO )

2004-05-09 Por tôpico Rafael
Graças ao Fábio D. Moreira, agora sabemos que a "lorota" foi parcial...

Como o problema, pelo visto, interessou a várias pessoas da lista, eis a
demonstração que eu havia omitido:

(x + y)^7 - x^7 - y^7 =
= 7x^6y + 21x^5.y^2 + 35x^4.y^3 + 35x^3.y^4 + 21x^2.y^5 + 7x.y^6
= 7xy(x^5 + 3x^4.y + 5x^3.y^2 + 5x^2.y^2 + 3x.y^4 + y^5)
= 7xy[(x+y)(x^4-x^3.y+x^2.y^2-x.y^3+y^4+3xy(x^3+y^3)+5x^2.y^2(x+y)]
= 7xy[(x+y)(x^4-x^3.y+x^2.y^2-x.y^3+y^4+3xy(x+y)(x^2-xy+y^2)+5x^2.y^2(x+y)]
= 7xy(x+y)[x^4-x^3.y+x^2.y^2-x.y^3+y^4+3xy(x^2-xy+y^2)+5x^2.y^2]
= 7xy(x+y)[x^4 - x^3.y - x.y^3 + y^4 + 3xy(x^2 + xy + y^2)]
= 7xy(x+y)[x(x^3 - y^3) - y(x^3 - y^3) + 3xy(x^2 + xy + y^2)]
= 7xy(x+y)[(x-y)(x^3 - y^3) + 3xy(x^2 + xy + y^2)]
= 7xy(x+y)[(x-y)(x-y)(x^2 + xy + y^2) + 3xy(x^2 + xy + y^2)]
= 7xy(x+y)(x^2 + xy + y^2)(x^2 - 2xy + y^2 + 3xy)
= 7xy(x+y)(x^2 + xy + y^2)^2


Abraços,

Rafael de A. Sampaio





- Original Message -
From: "Fabio Contreiras" <[EMAIL PROTECTED]>
To: <[EMAIL PROTECTED]>
Sent: Sunday, May 09, 2004 4:00 PM
Subject: [obm-l] Re: [obm-l] Re: [obm-l] Fatoração ( IMO )


Valeu rafael, po então foi lorota do cara que me passou isso :) abraços!



- Original Message -
From: "Rafael" <[EMAIL PROTECTED]>
To: "OBM-L" <[EMAIL PROTECTED]>
Sent: Sunday, May 09, 2004 2:55 PM
Subject: [obm-l] Re: [obm-l] Fatoração ( IMO )


> Fábio,
>
> Acho pouco provável que esse tipo de exercício tenha caído numa IMO,
mas...
>
> (x + y)^7 - (x^7 + y^7) = 7xy(x + y)(x^2 + xy + y^2)^2
>
> Uma identidade semelhante foi usada por Lamé na demonstração do Último
> Teorema de Fermat para n = 7.
>
> (x + y + z)^7 - (x^7 + y^7 + z^7) =
> = 7(x+y)(x+z)(y+z)[(x^2 + y^2 + z^2 + xy + xz + yz)^2 + xyz(x + y + z)]
>
>
> Abraços,
>
> Rafael de A. Sampaio





=
Instruções para entrar na lista, sair da lista e usar a lista em
http://www.mat.puc-rio.br/~nicolau/olimp/obm-l.html
=


Re: [obm-l] 8a.cone sul

2004-05-09 Por tôpico vieirausp
Em 8 May 2004, [EMAIL PROTECTED] escreveu: 

>a=15*t^2-15t-995 
>b=15*t^2-20t-992 
>c=15*t^2-18t-993 
>como chegar nestes valores de a,b e c.Estou com a eureka n7 em mãos e já tô 
correndo atrás, mas me de uma pista. 
> > 

_
Voce quer um iGMail protegido contra vírus e spams? 
Clique aqui: http://www.igmailseguro.ig.com.br
Ofertas imperdíveis! Link: http://www.americanas.com.br/ig/

=
Instruções para entrar na lista, sair da lista e usar a lista em
http://www.mat.puc-rio.br/~nicolau/olimp/obm-l.html
=


Re: [obm-l] 8a.cone sul

2004-05-09 Por tôpico vieirausp
Em 9 May 2004, [EMAIL PROTECTED] escreveu: 

>ta mto confuso o enunciado 
>tem certeza q transcreveu corretamente? 
>Sim, o enunciado está correto tal qual se encontra no site da obm(8a cone 
sul)- Original Message - 
>From: 
>To: ; 
>Sent: Sunday, May 09, 2004 4:11 PM 
>Subject: Re: [obm-l] 8a.cone sul 
> 
>> Em 8 May 2004, [EMAIL PROTECTED] escreveu: 
>> 
>> >Show de bola.Seja C uma circunferência de centro O, AB um diâmetro dela 
e 
>R 
>> >um ponto qualquer em C distinto de A e de B.Seja P a interseçâo da 
>> >perpendicular traçada por O a AR.Sobre a reta OP se marca o ponto Q,de 
>> >maneira que QP é a metade de PO e Q não pertence ao segmento OP.Por Q 
>traçamos 
>> a 
>> >paralela a AB que corta a reta AR em T.Chamamos de H o ponto de 
>interseção 
>> >das retas AQ e OT.Provar que H,R e B são colineares. 
>> > 
>> >_ 
>> >Voce quer um iGMail protegido contra vírus e spams? 
>> >Clique aqui: http://www.igmailseguro.ig.com.br 
>> >Ofertas imperdíveis! Link: http://www.americanas.com.br/ig/ 
>> > 
>> 
>= 
>> >Instruções para entrar na lista, sair da lista e usar a lista em 
>> >http://www.mat.puc-rio.br/~nicolau/olimp/obm-l.html 
>> 
>= 
>> > 
>> >-- 
>> 
>> _ 
>> Voce quer um iGMail protegido contra vírus e spams? 
>> Clique aqui: http://www.igmailseguro.ig.com.br 
>> Ofertas imperdíveis! Link: http://www.americanas.com.br/ig/ 
>> 
>> = 
>> Instruções para entrar na lista, sair da lista e usar a lista em 
>> http://www.mat.puc-rio.br/~nicolau/olimp/obm-l.html 
>> = 
>> 
>> 
> 
>= 
>Instruções para entrar na lista, sair da lista e usar a lista em 
>http://www.mat.puc-rio.br/~nicolau/olimp/obm-l.html 
>= 
> 
>-- 

_
Voce quer um iGMail protegido contra vírus e spams? 
Clique aqui: http://www.igmailseguro.ig.com.br
Ofertas imperdíveis! Link: http://www.americanas.com.br/ig/

=
Instruções para entrar na lista, sair da lista e usar a lista em
http://www.mat.puc-rio.br/~nicolau/olimp/obm-l.html
=


[obm-l] Re: [obm-l] Re:_[obm-l]_Princípio_de_Dirichlet

2004-05-09 Por tôpico Frederico Reis Marques de Brito
Lamentável perceber, novamente, que alguns particípantes da lista usam-na 
com fins diversos
dos quais ela foi criada. Concordo que o enunciado, tal como proposto, não é 
nenhum modelo de precisão, mas também acho que seu real significado é por 
demais evidente e que o objetivo era , tão somente, exemplificar o uso do 
Princípio das Gavetas num caso não muito trivial...
"Servaõ-lhe de lição os possíveis erros do passado... "

Frederico.

From: Johann Peter Gustav Lejeune Dirichlet 
<[EMAIL PROTECTED]>
Reply-To: [EMAIL PROTECTED]
To: [EMAIL PROTECTED]
Subject: Re: [obm-l] Re:_[obm-l]_Princípio_de_Dirichlet
Date: Sun, 9 May 2004 16:21:11 -0300 (ART)

Basta parti-lo ao meio!
--- Fabiano Sant'Ana <[EMAIL PROTECTED]>
escreveu: > como um simples ponto poderá possuir
Duas
> cores?
> - Original Message -
> From: "Frederico Reis Marques de Brito"
> <[EMAIL PROTECTED]>
> To: <[EMAIL PROTECTED]>
> Sent: Saturday, May 08, 2004 10:54 PM
> Subject: [obm-l] Princípio de Dirichlet
>
>
> > Mais um probleminha de contagem:
> >
> > Se pintarmos cada ponto de um círculo com
> duas cores, de forma aleatória,
> > então existirão
> > três pontos equidistantes pintados com a
> mesma cor.
> >
> >
> > Fred.
> >
> >
>
_
> > MSN Hotmail, o maior webmail do Brasil.
> http://www.hotmail.com
> >
> >
>
=
> > Instruções para entrar na lista, sair da
> lista e usar a lista em
> >
>
http://www.mat.puc-rio.br/~nicolau/olimp/obm-l.html
> >
>
=
> >
>
>
> ---
> Outgoing mail is certified Virus Free.
> Checked by AVG anti-virus system
> (http://www.grisoft.com).
> Version: 6.0.676 / Virus Database: 438 -
> Release Date: 04/05/04
>
>
=
> Instruções para entrar na lista, sair da lista
> e usar a lista em
>
http://www.mat.puc-rio.br/~nicolau/olimp/obm-l.html
>
=
=
TRANSIRE SVVM PECTVS MVNDOQVE POTIRI
CONGREGATI EX TOTO ORBE MATHEMATICI OB SCRIPTA INSIGNIA TRIBVERE
Fields Medal(John Charles Fields)
N.F.C. (Ne Fronti Crede)

__
Yahoo! Messenger - Fale com seus amigos online. Instale agora!
http://br.download.yahoo.com/messenger/
=
Instruções para entrar na lista, sair da lista e usar a lista em
http://www.mat.puc-rio.br/~nicolau/olimp/obm-l.html
=
_
MSN Messenger: converse com os seus amigos online.  
http://messenger.msn.com.br

=
Instruções para entrar na lista, sair da lista e usar a lista em
http://www.mat.puc-rio.br/~nicolau/olimp/obm-l.html
=


Re: [obm-l] 8a.cone sul

2004-05-09 Por tôpico vieirausp
Em 9 May 2004, [EMAIL PROTECTED] escreveu: 

>ta mto confuso o enunciado 
>tem certeza q transcreveu corretamente? 
>Sim, o enunciado está correto tal qual se encontra no arquivo de provas da 
obm(8a cone sul) 
>- Original Message - 
>From: 
>To: ; 
>Sent: Sunday, May 09, 2004 4:11 PM 
>Subject: Re: [obm-l] 8a.cone sul 
> 
>> Em 8 May 2004, [EMAIL PROTECTED] escreveu: 
>> 
>> >Show de bola.Seja C uma circunferência de centro O, AB um diâmetro dela 
e 
>R 
>> >um ponto qualquer em C distinto de A e de B.Seja P a interseçâo da 
>> >perpendicular traçada por O a AR.Sobre a reta OP se marca o ponto Q,de 
>> >maneira que QP é a metade de PO e Q não pertence ao segmento OP.Por Q 
>traçamos 
>> a 
>> >paralela a AB que corta a reta AR em T.Chamamos de H o ponto de 
>interseção 
>> >das retas AQ e OT.Provar que H,R e B são colineares. 
>> > 
>> >_ 
>> >Voce quer um iGMail protegido contra vírus e spams? 
>> >Clique aqui: http://www.igmailseguro.ig.com.br 
>> >Ofertas imperdíveis! Link: http://www.americanas.com.br/ig/ 
>> > 
>> 
>= 
>> >Instruções para entrar na lista, sair da lista e usar a lista em 
>> >http://www.mat.puc-rio.br/~nicolau/olimp/obm-l.html 
>> 
>= 
>> > 
>> >-- 
>> 
>> _ 
>> Voce quer um iGMail protegido contra vírus e spams? 
>> Clique aqui: http://www.igmailseguro.ig.com.br 
>> Ofertas imperdíveis! Link: http://www.americanas.com.br/ig/ 
>> 
>> = 
>> Instruções para entrar na lista, sair da lista e usar a lista em 
>> http://www.mat.puc-rio.br/~nicolau/olimp/obm-l.html 
>> = 
>> 
>> 
> 
>= 
>Instruções para entrar na lista, sair da lista e usar a lista em 
>http://www.mat.puc-rio.br/~nicolau/olimp/obm-l.html 
>= 
> 
>-- 

_
Voce quer um iGMail protegido contra vírus e spams? 
Clique aqui: http://www.igmailseguro.ig.com.br
Ofertas imperdíveis! Link: http://www.americanas.com.br/ig/

=
Instruções para entrar na lista, sair da lista e usar a lista em
http://www.mat.puc-rio.br/~nicolau/olimp/obm-l.html
=


Re: [obm-l] 8a.cone sul

2004-05-09 Por tôpico Fellipe Rossi
o treho q está confuso eh o da perpendicular a O.

no caso, P é a inteseeção da reta AR traçada por O (o "pé" da perpendicular)
ou é a interseção da reta perpendicular a AB por O?

eu creio q seja a primeira opção :)
- Original Message -
From: <[EMAIL PROTECTED]>
To: <[EMAIL PROTECTED]>; <[EMAIL PROTECTED]>
Sent: Sunday, May 09, 2004 4:11 PM
Subject: Re: [obm-l] 8a.cone sul


> Em 8 May 2004, [EMAIL PROTECTED] escreveu:
>
> >Show de bola.Seja C uma circunferência de centro O, AB um diâmetro dela e
R
> >um ponto qualquer em C distinto de A e de B.Seja P a interseçâo da
> >perpendicular traçada por O a AR.Sobre a reta OP se marca o ponto Q,de
> >maneira QP é a metade de PO e Q não pertence ao segmento OP.Por Q
traçamos
> a
> >paralela a AB que corta a reta AR em T.Chamamos de H o ponto de
interseção
> >das retas AQ e OT.Provar que H,R e B são colineares.
> >
> >_
> >Voce quer um iGMail protegido contra vírus e spams?
> >Clique aqui: http://www.igmailseguro.ig.com.br
> >Ofertas imperdíveis! Link: http://www.americanas.com.br/ig/
> >
> >=
> >Instruções para entrar na lista, sair da lista e usar a lista em
> >http://www.mat.puc-rio.br/~nicolau/olimp/obm-l.html
> >=
> >
> >--
>
> _
> Voce quer um iGMail protegido contra vírus e spams?
> Clique aqui: http://www.igmailseguro.ig.com.br
> Ofertas imperdíveis! Link: http://www.americanas.com.br/ig/
>
> =
> Instruções para entrar na lista, sair da lista e usar a lista em
> http://www.mat.puc-rio.br/~nicolau/olimp/obm-l.html
> =
>
>


=
Instruções para entrar na lista, sair da lista e usar a lista em
http://www.mat.puc-rio.br/~nicolau/olimp/obm-l.html
=


Re: [obm-l] 8a.cone sul

2004-05-09 Por tôpico Fellipe Rossi
ta mto confuso o enunciado
tem certeza q transcreveu corretamente?


- Original Message -
From: <[EMAIL PROTECTED]>
To: <[EMAIL PROTECTED]>; <[EMAIL PROTECTED]>
Sent: Sunday, May 09, 2004 4:11 PM
Subject: Re: [obm-l] 8a.cone sul


> Em 8 May 2004, [EMAIL PROTECTED] escreveu:
>
> >Show de bola.Seja C uma circunferência de centro O, AB um diâmetro dela e
R
> >um ponto qualquer em C distinto de A e de B.Seja P a interseçâo da
> >perpendicular traçada por O a AR.Sobre a reta OP se marca o ponto Q,de
> >maneira QP é a metade de PO e Q não pertence ao segmento OP.Por Q
traçamos
> a
> >paralela a AB que corta a reta AR em T.Chamamos de H o ponto de
interseção
> >das retas AQ e OT.Provar que H,R e B são colineares.
> >
> >_
> >Voce quer um iGMail protegido contra vírus e spams?
> >Clique aqui: http://www.igmailseguro.ig.com.br
> >Ofertas imperdíveis! Link: http://www.americanas.com.br/ig/
> >
> >=
> >Instruções para entrar na lista, sair da lista e usar a lista em
> >http://www.mat.puc-rio.br/~nicolau/olimp/obm-l.html
> >=
> >
> >--
>
> _
> Voce quer um iGMail protegido contra vírus e spams?
> Clique aqui: http://www.igmailseguro.ig.com.br
> Ofertas imperdíveis! Link: http://www.americanas.com.br/ig/
>
> =
> Instruções para entrar na lista, sair da lista e usar a lista em
> http://www.mat.puc-rio.br/~nicolau/olimp/obm-l.html
> =
>
>


=
Instruções para entrar na lista, sair da lista e usar a lista em
http://www.mat.puc-rio.br/~nicolau/olimp/obm-l.html
=


Re: [obm-l] Re: [obm-l] Re: [obm-l] Fatoração ( IMO )

2004-05-09 Por tôpico Fabio Dias Moreira

Fabio Contreiras said:
> Valeu rafael, po então foi lorota do cara que me passou isso :) abraços!
> [...]

Eu acho que você quer o seguinte problema:

(IMO-84) Encontre todos os inteiros a, b tais que ab(a+b) não é múltiplo
de 7 mas (a+b)^7 - (a^7 + b^7) é divisível por 7^7.

[]s,

-- 
Fábio "ctg \pi" Dias Moreira


=
Instruções para entrar na lista, sair da lista e usar a lista em
http://www.mat.puc-rio.br/~nicolau/olimp/obm-l.html
=


Re: [obm-l] Re:_[obm-l]_Re:_[obm-l]_Fatoração_(_IMO_)

2004-05-09 Por tôpico Johann Peter Gustav Lejeune Dirichlet
25a IMO, 1984, Problema 2
 --- Fabio Contreiras
<[EMAIL PROTECTED]> escreveu: > Valeu
rafael, po então foi lorota do cara que
> me passou isso :) abraços!
> - Original Message - 
> From: "Rafael" <[EMAIL PROTECTED]>
> To: "OBM-L" <[EMAIL PROTECTED]>
> Sent: Sunday, May 09, 2004 2:55 PM
> Subject: [obm-l] Re: [obm-l] Fatoração ( IMO )
> 
> 
> > Fábio,
> >
> > Acho pouco provável que esse tipo de
> exercício tenha caído numa IMO,
> mas...
> >
> > (x + y)^7 - (x^7 + y^7) = 7xy(x + y)(x^2 + xy
> + y^2)^2
> >
> > Uma identidade semelhante foi usada por Lamé
> na demonstração do Último
> > Teorema de Fermat para n = 7.
> >
> > (x + y + z)^7 - (x^7 + y^7 + z^7) =
> > = 7(x+y)(x+z)(y+z)[(x^2 + y^2 + z^2 + xy + xz
> + yz)^2 + xyz(x + y + z)]
> >
> >
> > Abraços,
> >
> > Rafael de A. Sampaio
> >
> >
> >
> >
> >
> > - Original Message -
> > From: Fabio Contreiras
> > To: [EMAIL PROTECTED]
> > Sent: Sunday, May 09, 2004 2:32 PM
> > Subject: [obm-l] Fatoração ( IMO )
> >
> >
> > Alguem tem ideia de como fatorar isso? Um
> Abraço!
> >
> >
> > ( x + y )^7 - ( x^7 + y^7 )
> >
> >
> >
> >
>
=
> > Instruções para entrar na lista, sair da
> lista e usar a lista em
> >
>
http://www.mat.puc-rio.br/~nicolau/olimp/obm-l.html
> >
>
=
> >
> 
>
=
> Instruções para entrar na lista, sair da lista
> e usar a lista em
>
http://www.mat.puc-rio.br/~nicolau/olimp/obm-l.html
>
= 

=
TRANSIRE SVVM PECTVS MVNDOQVE POTIRI 

CONGREGATI EX TOTO ORBE MATHEMATICI OB SCRIPTA INSIGNIA TRIBVERE 

Fields Medal(John Charles Fields)
 
N.F.C. (Ne Fronti Crede)



__

Yahoo! Messenger - Fale com seus amigos online. Instale agora! 
http://br.download.yahoo.com/messenger/
=
Instruções para entrar na lista, sair da lista e usar a lista em
http://www.mat.puc-rio.br/~nicolau/olimp/obm-l.html
=


Re: [obm-l] Retas que se cruzam

2004-05-09 Por tôpico Johann Peter Gustav Lejeune Dirichlet
A primeira pergunta que eu faria e:
O que e ponto, reta e cruzar?No sentido de quais
sao os axiomas?


 --- Ricardo Bittencourt <[EMAIL PROTECTED]>
escreveu: > Robério Alves wrote:
> > Em quantos pontos duas retas se cruzam ? 
> Analise de todas as formas 
> > possíveis .
> 
>   Em qual geometria? Na Euclideana plana
> clássica,
> as respostas possíveis são 1 ponto (retas
> concorrentes),
> nenhum ponto (retas paralelas), ou todos os
> pontos
> (retas coincidentes).
> 
>

> Ricardo Bittencourt  
> http://www.mundobizarro.tk
> [EMAIL PROTECTED]   "tenki ga ii kara
> sanpo shimashou"
> -- União contra o forward - crie suas
> proprias piadas --
>
=
> Instruções para entrar na lista, sair da lista
> e usar a lista em
>
http://www.mat.puc-rio.br/~nicolau/olimp/obm-l.html
>
= 

=
TRANSIRE SVVM PECTVS MVNDOQVE POTIRI 

CONGREGATI EX TOTO ORBE MATHEMATICI OB SCRIPTA INSIGNIA TRIBVERE 

Fields Medal(John Charles Fields)
 
N.F.C. (Ne Fronti Crede)



__

Yahoo! Messenger - Fale com seus amigos online. Instale agora! 
http://br.download.yahoo.com/messenger/
=
Instruções para entrar na lista, sair da lista e usar a lista em
http://www.mat.puc-rio.br/~nicolau/olimp/obm-l.html
=


Re: [obm-l] Re:_[obm-l]_Princípio_de_Dirichlet

2004-05-09 Por tôpico Johann Peter Gustav Lejeune Dirichlet
Basta parti-lo ao meio!
--- Fabiano Sant'Ana <[EMAIL PROTECTED]>
escreveu: > como um simples ponto poderá possuir
Duas
> cores?
> - Original Message -
> From: "Frederico Reis Marques de Brito"
> <[EMAIL PROTECTED]>
> To: <[EMAIL PROTECTED]>
> Sent: Saturday, May 08, 2004 10:54 PM
> Subject: [obm-l] Princípio de Dirichlet
> 
> 
> > Mais um probleminha de contagem:
> >
> > Se pintarmos cada ponto de um círculo com
> duas cores, de forma aleatória,
> > então existirão
> > três pontos equidistantes pintados com a
> mesma cor.
> >
> >
> > Fred.
> >
> >
>
_
> > MSN Hotmail, o maior webmail do Brasil. 
> http://www.hotmail.com
> >
> >
>
=
> > Instruções para entrar na lista, sair da
> lista e usar a lista em
> >
>
http://www.mat.puc-rio.br/~nicolau/olimp/obm-l.html
> >
>
=
> >
> 
> 
> ---
> Outgoing mail is certified Virus Free.
> Checked by AVG anti-virus system
> (http://www.grisoft.com).
> Version: 6.0.676 / Virus Database: 438 -
> Release Date: 04/05/04
> 
>
=
> Instruções para entrar na lista, sair da lista
> e usar a lista em
>
http://www.mat.puc-rio.br/~nicolau/olimp/obm-l.html
>
= 

=
TRANSIRE SVVM PECTVS MVNDOQVE POTIRI 

CONGREGATI EX TOTO ORBE MATHEMATICI OB SCRIPTA INSIGNIA TRIBVERE 

Fields Medal(John Charles Fields)
 
N.F.C. (Ne Fronti Crede)



__

Yahoo! Messenger - Fale com seus amigos online. Instale agora! 
http://br.download.yahoo.com/messenger/
=
Instruções para entrar na lista, sair da lista e usar a lista em
http://www.mat.puc-rio.br/~nicolau/olimp/obm-l.html
=


Re: [obm-l] Fatoração ( IMO )

2004-05-09 Por tôpico Johann Peter Gustav Lejeune Dirichlet
O comum na fatoraçao e fazer os termos serem dois
a dois coprimos.
 --- Eduardo Henrique Leitner
<[EMAIL PROTECTED]> escreveu: > = (x + y)^7 - (x
+ y)(x^6 - x^5y + x^4y^2 -
> x^3y^3 + x^2y^4 - xy^5 + y^6) =
> = (x+y)[ (x + y)^6 - (x^6 + y^6 -x^5y - xy^5 +
> x^4y^2 + x^2y^4 - x^3y^3) ]
> 
> aqui tenho uma duvida
> 
> o que exatamente significa fatorar? c eh
> colocar a expressão como sendo o produto de 2
> fatores, essa resposta jah eh valida...
> 
> On Sun, May 09, 2004 at 02:32:34PM -0300, Fabio
> Contreiras wrote:
> > Alguem tem ideia de como fatorar isso? Um
> Abraço!
> > 
> > 
> > ( x + y )^7 - ( x^7 + y^7 )
> 
>
=
> Instruções para entrar na lista, sair da lista
> e usar a lista em
>
http://www.mat.puc-rio.br/~nicolau/olimp/obm-l.html
>
= 

=
TRANSIRE SVVM PECTVS MVNDOQVE POTIRI 

CONGREGATI EX TOTO ORBE MATHEMATICI OB SCRIPTA INSIGNIA TRIBVERE 

Fields Medal(John Charles Fields)
 
N.F.C. (Ne Fronti Crede)



__

Yahoo! Messenger - Fale com seus amigos online. Instale agora! 
http://br.download.yahoo.com/messenger/
=
Instruções para entrar na lista, sair da lista e usar a lista em
http://www.mat.puc-rio.br/~nicolau/olimp/obm-l.html
=


Re: [obm-l] Re:_[obm-l]_Fatoração_(_IMO_)

2004-05-09 Por tôpico Johann Peter Gustav Lejeune Dirichlet
E, isso caiu na IMO mas nao desse jeito...A IMO
nunca mais faria perguntas nesse estilo.
Era algo como achar x e y com a propriedade de
essa expressao ser multipla de 7^7...


--- Rafael <[EMAIL PROTECTED]> escreveu: >
Fábio,
> 
> Acho pouco provável que esse tipo de exercício
> tenha caído numa IMO, mas...
> 
> (x + y)^7 - (x^7 + y^7) = 7xy(x + y)(x^2 + xy +
> y^2)^2
> 
> Uma identidade semelhante foi usada por Lamé na
> demonstração do Último
> Teorema de Fermat para n = 7.
> 
> (x + y + z)^7 - (x^7 + y^7 + z^7) =
> = 7(x+y)(x+z)(y+z)[(x^2 + y^2 + z^2 + xy + xz +
> yz)^2 + xyz(x + y + z)]
> 
> 
> Abraços,
> 
> Rafael de A. Sampaio
> 
> 
> 
> 
> 
> - Original Message -
> From: Fabio Contreiras
> To: [EMAIL PROTECTED]
> Sent: Sunday, May 09, 2004 2:32 PM
> Subject: [obm-l] Fatoração ( IMO )
> 
> 
> Alguem tem ideia de como fatorar isso? Um
> Abraço!
> 
> 
> ( x + y )^7 - ( x^7 + y^7 )
> 
> 
> 
>
=
> Instruções para entrar na lista, sair da lista
> e usar a lista em
>
http://www.mat.puc-rio.br/~nicolau/olimp/obm-l.html
>
= 

=
TRANSIRE SVVM PECTVS MVNDOQVE POTIRI 

CONGREGATI EX TOTO ORBE MATHEMATICI OB SCRIPTA INSIGNIA TRIBVERE 

Fields Medal(John Charles Fields)
 
N.F.C. (Ne Fronti Crede)



__

Yahoo! Messenger - Fale com seus amigos online. Instale agora! 
http://br.download.yahoo.com/messenger/
=
Instruções para entrar na lista, sair da lista e usar a lista em
http://www.mat.puc-rio.br/~nicolau/olimp/obm-l.html
=


Re: [obm-l] 8a.cone sul

2004-05-09 Por tôpico vieirausp
Em 8 May 2004, [EMAIL PROTECTED] escreveu: 

>Show de bola.Seja C uma circunferência de centro O, AB um diâmetro dela e R 
>um ponto qualquer em C distinto de A e de B.Seja P a interseçâo da 
>perpendicular traçada por O a AR.Sobre a reta OP se marca o ponto Q,de 
>maneira QP é a metade de PO e Q não pertence ao segmento OP.Por Q traçamos 
a 
>paralela a AB que corta a reta AR em T.Chamamos de H o ponto de interseção 
>das retas AQ e OT.Provar que H,R e B são colineares. 
> 
>_ 
>Voce quer um iGMail protegido contra vírus e spams? 
>Clique aqui: http://www.igmailseguro.ig.com.br 
>Ofertas imperdíveis! Link: http://www.americanas.com.br/ig/ 
> 
>= 
>Instruções para entrar na lista, sair da lista e usar a lista em 
>http://www.mat.puc-rio.br/~nicolau/olimp/obm-l.html 
>= 
> 
>-- 

_
Voce quer um iGMail protegido contra vírus e spams? 
Clique aqui: http://www.igmailseguro.ig.com.br
Ofertas imperdíveis! Link: http://www.americanas.com.br/ig/

=
Instruções para entrar na lista, sair da lista e usar a lista em
http://www.mat.puc-rio.br/~nicolau/olimp/obm-l.html
=


Re: [obm-l] 8a.cone sul

2004-05-09 Por tôpico vieirausp
Em 8 May 2004, [EMAIL PROTECTED] escreveu: 

>Show de bola.Seja C uma circunferência de centro O, AB um diâmetro dela e R 
>um ponto qualquer em C distinto de A e de B.Seja P a interseçâo da 
>perpendicular traçada por O a AR.Sobre a reta OP se marca o ponto Q,de 
>maneira QP é a metade de PO e Q não pertence ao segmento OP.Por Q traçamos 
a 
>paralela a AB que corta a reta AR em T.Chamamos de H o ponto de interseção 
>das retas AQ e OT.Provar que H,R e B são colineares. 
> 
>_ 
>Voce quer um iGMail protegido contra vírus e spams? 
>Clique aqui: http://www.igmailseguro.ig.com.br 
>Ofertas imperdíveis! Link: http://www.americanas.com.br/ig/ 
> 
>= 
>Instruções para entrar na lista, sair da lista e usar a lista em 
>http://www.mat.puc-rio.br/~nicolau/olimp/obm-l.html 
>= 
> 
>-- 

_
Voce quer um iGMail protegido contra vírus e spams? 
Clique aqui: http://www.igmailseguro.ig.com.br
Ofertas imperdíveis! Link: http://www.americanas.com.br/ig/

=
Instruções para entrar na lista, sair da lista e usar a lista em
http://www.mat.puc-rio.br/~nicolau/olimp/obm-l.html
=


[obm-l] Re: [obm-l] Re: [obm-l] Fatoração ( IMO )

2004-05-09 Por tôpico Fabio Contreiras
Valeu rafael, po então foi lorota do cara que me passou isso :) abraços!
- Original Message - 
From: "Rafael" <[EMAIL PROTECTED]>
To: "OBM-L" <[EMAIL PROTECTED]>
Sent: Sunday, May 09, 2004 2:55 PM
Subject: [obm-l] Re: [obm-l] Fatoração ( IMO )


> Fábio,
>
> Acho pouco provável que esse tipo de exercício tenha caído numa IMO,
mas...
>
> (x + y)^7 - (x^7 + y^7) = 7xy(x + y)(x^2 + xy + y^2)^2
>
> Uma identidade semelhante foi usada por Lamé na demonstração do Último
> Teorema de Fermat para n = 7.
>
> (x + y + z)^7 - (x^7 + y^7 + z^7) =
> = 7(x+y)(x+z)(y+z)[(x^2 + y^2 + z^2 + xy + xz + yz)^2 + xyz(x + y + z)]
>
>
> Abraços,
>
> Rafael de A. Sampaio
>
>
>
>
>
> - Original Message -
> From: Fabio Contreiras
> To: [EMAIL PROTECTED]
> Sent: Sunday, May 09, 2004 2:32 PM
> Subject: [obm-l] Fatoração ( IMO )
>
>
> Alguem tem ideia de como fatorar isso? Um Abraço!
>
>
> ( x + y )^7 - ( x^7 + y^7 )
>
>
>
> =
> Instruções para entrar na lista, sair da lista e usar a lista em
> http://www.mat.puc-rio.br/~nicolau/olimp/obm-l.html
> =
>

=
Instruções para entrar na lista, sair da lista e usar a lista em
http://www.mat.puc-rio.br/~nicolau/olimp/obm-l.html
=


[obm-l] Princípio de Dirichlet - variacao

2004-05-09 Por tôpico Claudio Buffara
Frederico Reis Marques de Brito wrote:
> 
> Se pintarmos cada ponto de um círculo com [uma dentre] duas cores,
> de forma aleatória, então existirão
> três pontos equidistantes pintados com a mesma cor.
>
E se ao inves de circulo (ou seja, disco) o enunciado falasse em
circunferencia (de modo que nao pudessemos usar o centro)?
Ainda teriamos um triangulo equilatero com os 3 vertices da mesma cor?

[]s,
Claudio.


=
Instruções para entrar na lista, sair da lista e usar a lista em
http://www.mat.puc-rio.br/~nicolau/olimp/obm-l.html
=


[obm-l] Re:[obm-l] Fatoração ( IMO )

2004-05-09 Por tôpico Osvaldo
> Alguem tem ideia de como fatorar isso? Um Abraço!
> 
> 
> ( x + y )^7 - ( x^7 + y^7 )


Basta desenvolver o Binômio de Newton...
vão se cancelar o primeiro e o último termos.
Depois basta colocar (x.y) em evidencia.





Atenciosamente,

Engenharia Elétrica - UNESP Ilha Solteira
Osvaldo Mello Sponquiado 
Usuário de GNU/Linux


 
__
Acabe com aquelas janelinhas que pulam na sua tela.
AntiPop-up UOL - É grátis!
http://antipopup.uol.com.br/



=
Instruções para entrar na lista, sair da lista e usar a lista em
http://www.mat.puc-rio.br/~nicolau/olimp/obm-l.html
=


Re: [obm-l] Fatoração ( IMO )

2004-05-09 Por tôpico Eduardo Henrique Leitner
= (x + y)^7 - (x + y)(x^6 - x^5y + x^4y^2 - x^3y^3 + x^2y^4 - xy^5 + y^6) =
= (x+y)[ (x + y)^6 - (x^6 + y^6 -x^5y - xy^5 + x^4y^2 + x^2y^4 - x^3y^3) ]

aqui tenho uma duvida

o que exatamente significa fatorar? c eh colocar a expressão como sendo o produto de 2 
fatores, essa resposta jah eh valida...

On Sun, May 09, 2004 at 02:32:34PM -0300, Fabio Contreiras wrote:
> Alguem tem ideia de como fatorar isso? Um Abraço!
> 
> 
> ( x + y )^7 - ( x^7 + y^7 )

=
Instruções para entrar na lista, sair da lista e usar a lista em
http://www.mat.puc-rio.br/~nicolau/olimp/obm-l.html
=


RE: [obm-l] Fatoração ( IMO )

2004-05-09 Por tôpico Marcelo Souza


>From: "Fabio Contreiras" <[EMAIL PROTECTED]> 
>Reply-To: [EMAIL PROTECTED] 
>To: <[EMAIL PROTECTED]> 
>Subject: [obm-l] Fatoração ( IMO ) 
>Date: Sun, 9 May 2004 14:32:34 -0300 
> 
>Alguem tem ideia de como fatorar isso? Um Abraço! 
> 
> 

>( x + y )^7 - ( x^7 + y^7 ) 
x^7+y^7=(x+y)(x^6-x^5y+x^4y^2-...-xy^5+y^6)
Daí você põe (x+y) em evidência pra obter:
(x+y)^7-x^7-y^7=(x+y)[(x+y)^6-x^6+x^5y-x^4y^2+...+xy^5-y^6)]
A rigor, já está fatorado, mas, você pode "ajeitar" a expressão dentro do [].
[]'s, Marcelo. 
MSN Messenger: converse com os seus amigos online. Instale grátis. Clique aqui. 
=
Instruções para entrar na lista, sair da lista e usar a lista em
http://www.mat.puc-rio.br/~nicolau/olimp/obm-l.html
=


[obm-l] Re: [obm-l] Fatoração ( IMO )

2004-05-09 Por tôpico Rafael
Fábio,

Acho pouco provável que esse tipo de exercício tenha caído numa IMO, mas...

(x + y)^7 - (x^7 + y^7) = 7xy(x + y)(x^2 + xy + y^2)^2

Uma identidade semelhante foi usada por Lamé na demonstração do Último
Teorema de Fermat para n = 7.

(x + y + z)^7 - (x^7 + y^7 + z^7) =
= 7(x+y)(x+z)(y+z)[(x^2 + y^2 + z^2 + xy + xz + yz)^2 + xyz(x + y + z)]


Abraços,

Rafael de A. Sampaio





- Original Message -
From: Fabio Contreiras
To: [EMAIL PROTECTED]
Sent: Sunday, May 09, 2004 2:32 PM
Subject: [obm-l] Fatoração ( IMO )


Alguem tem ideia de como fatorar isso? Um Abraço!


( x + y )^7 - ( x^7 + y^7 )



=
Instruções para entrar na lista, sair da lista e usar a lista em
http://www.mat.puc-rio.br/~nicolau/olimp/obm-l.html
=


[obm-l] Fatoração ( IMO )

2004-05-09 Por tôpico Fabio Contreiras



Alguem tem ideia de como fatorar isso? Um 
Abraço!
 
 
( x + y )^7 - ( x^7 + y^7 
)


[obm-l] Cadê?

2004-05-09 Por tôpico Fabio Henrique
Por onde andará Putinha da Silva? 

_
Voce quer um iGMail protegido contra vírus e spams? 
Clique aqui: http://www.igmailseguro.ig.com.br



Re: [obm-l] COLEGIO NAVAL

2004-05-09 Por tôpico Fabio Henrique
Continuando... 

Há um problema que pede o lado em função do raio R e a resposta é 
L.sqrt(2)/2. 

Em 8 May 2004, [EMAIL PROTECTED] escreveu: 

>leandro-epcar said: 
>> COLÉGIO NAVAL (1987) 
>> 
>> A equação do segundo grau X^2-2X+M=0,m<0,tem raízes X' 
>> e X" ,se X'^(N-2)+X"^(N-2)=A e X'^(N-1)+X"^(N-1) 
>> =B,então X'+X" é: 
>> (A) 2A+MB 
>> (B) 2B-MA 
>> (C) MA+2B 
>> (D) MA-2B 
>> (E) M(A-2B) 
>> [...] 
> 
>Eu suponho que você quer X'^N + X"^N? Caso contrário, é fácil ver que não 
>há solução. Neste caso, se X e Y são as raízes (para simplificar a 
>notação), temos que (x+y)(x^(n-1)+y^(n-1)) = x^n + y^n + 
>xy(x^(n-2)+y^(n-2)), mas a gente sabe quanto valem x+y e xy. 
> 
>[]s, 
> 
>-- 
>Fábio "ctg \pi" Dias Moreira 
> 
>= 
>Instruções para entrar na lista, sair da lista e usar a lista em 
>http://www.mat.puc-rio.br/~nicolau/olimp/obm-l.html 
>= 
> 
>-- 

_
Voce quer um iGMail protegido contra vírus e spams? 
Clique aqui: http://www.igmailseguro.ig.com.br



[obm-l] Re: [obm-l] dúvida

2004-05-09 Por tôpico Fellipe Rossi



cada corte aparecem 2 novos pedaços
logo pra termos catorze precisamos de 7 
cortes
como ele ja fez 1, serão necessários mais 
6
o ângulo do setor será 360 graus / 14 = 
aproximadamente 26 graus

  - Original Message - 
  From: 
  TSD 
  To: [EMAIL PROTECTED] 
  Sent: Saturday, May 08, 2004 6:35 
PM
  Subject: [obm-l] dúvida
  
  
  17 - Uma pizza de 
  formato circular será cortada em fatias pormeio de cortes quesão segmentos de 
  reta que passam pelo"centro" da pizza e medem um diâmetro. A figura a seguir 
  mostra que, com um corte, a pizza fica dividida em duas fatias iguais.( é uma 
  circunferencia com um traço diametro)
  Para se obter quatorze fatias iguais, a partir do 
  primeiro corte,serão necessários cortes adicionais na pizza. Assinale o item 
  que indica o número de cortes adicionais que terão de ser feitos e o ângulo 
  aproximado que cada fatia, imaginada como um setor circular, terá:
  (A) 8 e 30o;(B) 7 e 26o ;(C) 6 e 25o;(D) 7 e 30o; (E) 8 e 26o.
   
   


Re: [obm-l] COLEGIO NAVAL

2004-05-09 Por tôpico Fabio Henrique
Sinceramente, não consigo entender os enunciados. 

Na questão abaixo, quem é N? É um natural qualquer? 

1/M^3 + 1/N^2? 


Em 8 May 2004, [EMAIL PROTECTED] escreveu: 

>leandro-epcar said: 
>> COLÉGIO NAVAL (1987) 
>> 
>> A equação do segundo grau X^2-2X+M=0,m<0,tem raízes X' 
>> e X" ,se X'^(N-2)+X"^(N-2)=A e X'^(N-1)+X"^(N-1) 
>> =B,então X'+X" é: 
>> (A) 2A+MB 
>> (B) 2B-MA 
>> (C) MA+2B 
>> (D) MA-2B 
>> (E) M(A-2B) 
>> [...] 
> 
>Eu suponho que você quer X'^N + X"^N? Caso contrário, é fácil ver que não 
>há solução. Neste caso, se X e Y são as raízes (para simplificar a 
>notação), temos que (x+y)(x^(n-1)+y^(n-1)) = x^n + y^n + 
>xy(x^(n-2)+y^(n-2)), mas a gente sabe quanto valem x+y e xy. 
> 
>[]s, 
> 
>-- 
>Fábio "ctg \pi" Dias Moreira 
> 
>= 
>Instruções para entrar na lista, sair da lista e usar a lista em 
>http://www.mat.puc-rio.br/~nicolau/olimp/obm-l.html 
>= 
> 
>-- 

_
Voce quer um iGMail protegido contra vírus e spams? 
Clique aqui: http://www.igmailseguro.ig.com.br



Re: [obm-l] cococolegio navalvalval

2004-05-09 Por tôpico Fabio Henrique
Há um erro no final da solução ou estou equivocado? 
Se M^3=1000-3(10)-N^3, então M^3+N^3=970. 

Em  8 May 2004, [EMAIL PROTECTED] escreveu: 

>Eu sei que estou sendo incoveniente mas essas 
>quetões que eu venho mandando são duvidas de acumuladas 
>de 2 anos,ficarei muito grato se voçês me ajudarem. 
> 
> COLÉGIO NAVAL (1989) 
> 
> Sendo M e N as raízes da equação X^2-10X+1=0 , o 
>valor da expressão 1/M^3 + 1/N^2 é : 
> 
> (A) 970 
> (B) 950 
> (C) 920 
> (D) 900 
> (E) 870 
>=== 
> OBSERVAÇÃO: 
> 
> Fazendo as raízes da equação ,encontraremos {5-2[6^ 
>(1/2)]} e {5+2[6^(1/2)]} 
> 
> fazendo de acordo com o enunciado teremos: 
> 
> 1 + 1 
>  = 485+198(6^(1/2)) 
>{5-2[6^(1/2)]}^3} {5+2[6^(1/2)]}^2} 
> 
> Oque se aproxíma de 970,mas a resposta não é 
>aproximada ,é exata e se voçê inverter os valores de M 
>e N, o resultado será difernte. 
>== 
> Fiz uma anologia más não ajudou muito 
> 
> (M+N)^3=M^3+N^3+3(M^2)N+3(N^2)M 
> 
> o que se conclui que : 
> 
> M^3=(M+N)^3-3MN(M+N)-N^3 
> 
> o que se conclui que: 
> 
> M^3=1000-3(10)-N^3 
> M^3-n^3=970 
> O QUE SE TORNOU ESTRANHO É CONCLUIR QUE : 
> 
> 1/(m^3)+1/(n^2) = m^3-n^3 
>=== 
> 
>__ 
>Acabe com aquelas janelinhas que pulam na sua tela. 
>AntiPop-up UOL - É grátis! 
>http://antipopup.uol.com.br/ 
> 
>= 
>Instruções para entrar na lista, sair da lista e usar a lista em 
>http://www.mat.puc-rio.br/~nicolau/olimp/obm-l.html 
>= 
> 
>-- 

_
Voce quer um iGMail protegido contra vírus e spams? 
Clique aqui: http://www.igmailseguro.ig.com.br



Re: [obm-l] Re: [obm-l] Princípio_de_Dirich let

2004-05-09 Por tôpico Fabio Henrique
Vocês não acham que o enunciado era para ser: "utilizando-se duas cores"? 


Em 09 May 2004, [EMAIL PROTECTED] escreveu: 

>Fabiano Sant'Ana wrote: 
> 
>> como um simples ponto poderá possuir Duas cores? 
> 
> Desenha um círculo no papel, aí pra cada ponto 
>do círculo você pinta de azul por cima da folha, e de 
>vermelho na parte de baixo. Pronto, agora um simples 
>ponto tem duas cores! 
> 
> Eu aqui estou trabalhando profissionalmente 
>com coisa muito mais bizarra, que é ponto orientado 
>(na verdade, vetor 2D degenerado cuja magnitude foi 
>pra zero, mas preservou a direção e sentido). 
> 
> 
>Ricardo Bittencourt http://www.mundobizarro.tk 
>[EMAIL PROTECTED] "tenki ga ii kara sanpo shimashou" 
>-- União contra o forward - crie suas proprias piadas -- 
>= 
>Instruções para entrar na lista, sair da lista e usar a lista em 
>http://www.mat.puc-rio.br/~nicolau/olimp/obm-l.html 
>= 
> 
>-- 

_
Voce quer um iGMail protegido contra vírus e spams? 
Clique aqui: http://www.igmailseguro.ig.com.br



RE: [obm-l] En:COLEGIO NAVAL

2004-05-09 Por tôpico Rogério Moraes de Carvalho
Olá Leandro,

Seria interessante que você já disponibilizasse o mais rápido
possível as provas que você tem do Colégio Naval para os participantes da
lista. Deste modo, você estaria facilitando o acesso às questões originais
por parte de todos e ficaria mais fácil poder ajudá-lo.

Abraços,

Rogério Moraes de Carvalho
[EMAIL PROTECTED]


-Original Message-
From: [EMAIL PROTECTED] [mailto:[EMAIL PROTECTED] On
Behalf Of leandro-epcar
Sent: sábado, 8 de maio de 2004 15:49
To: obm-l
Subject: Re: [obm-l] En:COLEGIO NAVAL

   Em breve colocarei as provas do colegio naval na 
internet.
   Sobre "" LEANDRO-EPCAR"" eu diria em rumo À ""EPCAR""
 ou o colegio naval.
   Esse ano vou fazer a prova e pretendo passar.

   ATENCIOSAMENTE

   LEANDRO 

-- Início da mensagem original ---

  De: "Alan Pellejero" 
[EMAIL PROTECTED]
Para: [EMAIL PROTECTED]
  Cc: 
Data: Thu, 6 May 2004 18:42:58 -0300 (ART)
 Assunto: Re: [obm-l] En:COLEGIO NAVAL

> e ai cara, meu, eu ficaria agradecido se vc me 
mandasse
> vc faz mesmo epcar ou esse nick é só pra botar uma 
moral???
> hehe
> valeu cara,
> té mais!
> Alan Pellejero
> 
> leandro-epcar <[EMAIL PROTECTED]> wrote:
> -- Início da mensagem original ---
> 
> De: "leandro-epcar" leandro-
> [EMAIL PROTECTED]
> Para: "obm-l" [EMAIL PROTECTED]
> Cc: 
> Data: Wed, 28 Apr 2004 10:31:09 -0300
> Assunto: COLEGIO NAVAL
> 
> de:leandro geraldo da costa.
> para:obm-l.mat.puc-rio.com.br
> assunto:geometria plana.
> data: 28 de abril de 2004.
> -- 
> 
> PROVA DO COLEGIO NAVAL DE 2003.
> 
> Num quadrilátero ABCD tem-se:AB=42, BC=48, CD=64,
> DA=49 e P é o ponto de interseção entre as diagonais
> AC e BD .Qual a razão entre os segmentos PA e PC ,sa-
> bendo-se que a diagonal BD é igual a 56.
> 
> (A)7/8.
> (B)8/7.
> (C)7/6.
> (D)6/7.
> (E)49/64.
> --
> OBSERVAÇÃO:
> 
> Desenhando com escala notei que a diagonal BD é 
> bissetriz de ADC, mas não tenho como provar. 
> 
> --
> 
> Tenho as provas de admissao do colegio naval de
> 52 a 66 e 87 a 2003.
> se houver algum interesado ,entrem em contato . 
> 
> 

> __
> Acabe com aquelas janelinhas que pulam na sua tela.
> AntiPop-up UOL - É grátis!
> http://antipopup.uol.com.br/
> 
> 
> 
> 

__
> Acabe com aquelas janelinhas que pulam na sua tela.
> AntiPop-up UOL - É grátis!
> http://antipopup.uol.com.br/
> 
> 
> 
> 

=
> Instruções para entrar na lista, sair da lista e usar 
a lista em
> http://www.mat.puc-rio.br/~nicolau/olimp/obm-l.html
> 

=
> 
> 
> -
> Yahoo! Messenger - Fale com seus amigos online. 
Instale agora!
 
__
Acabe com aquelas janelinhas que pulam na sua tela.
AntiPop-up UOL - É grátis!
http://antipopup.uol.com.br/



=
Instruções para entrar na lista, sair da lista e usar a lista em
http://www.mat.puc-rio.br/~nicolau/olimp/obm-l.html
=




=
Instruções para entrar na lista, sair da lista e usar a lista em
http://www.mat.puc-rio.br/~nicolau/olimp/obm-l.html
=


RE: [obm-l] cococolegio navalvalval

2004-05-09 Por tôpico Paulo Santa Rita
Ola Pessoal,

Bendito erro de transcricao ! A solucao de um problema trivial acrescenta 
muito pouco ao
espirito e objetivos originais desta nossa lista.

Sem duvida e verdade que devemos apresentar os enunciados tal como eles 
realmente sao,
mas o erro de transcricao abaixo abaixo parece sugerir um problema MAIS 
CONFORME o elan
olimpico que nos inspira ...

Se M e N sao raizes da equacao X^2 - Bx + 1 = 0, B > 2, entao para P primo, 
existe uma forma
sintetica ( em funcao de B ) de exprimir

f(M,N) = 1/(M^(P+1))  +  1/(N^P)  ?

Se P=2, entao, claramente :

(M+N)^2 = M^2 + 2MN + N^2 = M^2 + N^2 + 2, pois MN = 1
(M+N)^3 = M^3 + 2(M^2)N + 3M(N^2) +N^2 = M^3 +3MN(M+N) + N^3
(M+N)^2 + (M+N)^3 - 3MN(M+N) - 2 = (M^2 + N^3) + (M^3 + N^2)
No caso, M=1/N e M^P + 1/M^P =f(M+1/M) e M+1/M = B para todo P ...

Um Abraco a Todos
Paulo Santa Rita
1,1222,090504
-Original Message-
From: [EMAIL PROTECTED] [mailto:[EMAIL PROTECTED] On
Behalf Of Pacini bores
Sent: sábado, 8 de maio de 2004 18:49
To: [EMAIL PROTECTED]
Subject: Re: [obm-l] cococolegio navalvalval
Olá  Leandro ,

O enunciado  correto eh :
  Sendo M e N as raízes da equação X^2-10X+1=0 , o
valor da expressão  1/M^3 + 1/N^3 ,  ok ?
[]´s Pacini



> COLÉGIO NAVAL (1989)
>
>   Sendo M e N as raízes da equação X^2-10X+1=0 , o
>valor da expressão  1/M^3 + 1/N^2 é :
>
>  (A) 970
>  (B) 950
>  (C) 920
>  (D) 900
>  (E) 870
_
MSN Messenger: converse com os seus amigos online.  
http://messenger.msn.com.br

=
Instruções para entrar na lista, sair da lista e usar a lista em
http://www.mat.puc-rio.br/~nicolau/olimp/obm-l.html
=


[obm-l] Princípio de Dirichlet - Outro

2004-05-09 Por tôpico Claudio Buffara
E aqui vai outro na mesma linha:

Se pintarmos cada ponto do plano de vermelho ou azul, entao existirah um
retangulo com os quatro vertices da mesma cor.

[]s,
Claudio.

on 08.05.04 22:54, Frederico Reis Marques de Brito at [EMAIL PROTECTED]
wrote:

> Mais um probleminha de contagem:
> 
> Se pintarmos cada ponto de um círculo com duas cores, de forma aleatória,
> então existirão
> três pontos equidistantes pintados com a mesma cor.
> 
> 
> Fred.
> 


=
Instruções para entrar na lista, sair da lista e usar a lista em
http://www.mat.puc-rio.br/~nicolau/olimp/obm-l.html
=


Re: [obm-l] Probabilidade e quadradinhos

2004-05-09 Por tôpico Claudio Buffara
Eu acho que eh assim:

Numero de casos possiveis =
Numero de maneiras de se escolher 2 quadrados =
Binom(n^2,2) = n^2*(n+1)*(n-1)/2

Numero de casos favoraveis =
Numero de maneiras de se escolher dois quadrados com um lado em comum =
Numero de maneiras de se escolher um "domino" =
2*n*(n-1)

Probabilidade = 2*n*(n-1)/((n^2*(n+1)*(n-1)/2) = 4/(n*(n+1))


[]s,
Claudio.

on 09.05.04 04:09, Rafael at [EMAIL PROTECTED] wrote:

> Pessoal,
> 
> Em outra lista, vi um problema interessante:
> 
> Uma folha quadrada de papel quadriculado contém n^2 quadradinhos (n >= 2).
> Escolhendo-se, ao acaso, dois quadradinhos distintos, qual é a probabilidade
> de que eles tenham um lado comum?
> 
> ..
> 
> 
> Há três "tipos" de quadradinhos: a) aqueles que estão nos cantos da folha e
> possuem apenas dois outros quadradinhos adjacentes; b) aqueles que estão ao
> longo das extremidades da folha e possuem apenas três quadradinhos vizinhos;
> e c) aqueles que não estão em qualquer uma das posições anteriores da folha
> e possuem quatro quadradinhos vizinhos.
> 
> Qualquer que seja n >= 2, teremos para:
> 
> a) 4 quadradinhos;
> b) 4(n-2) quadradinhos;
> c) n^2 - 4 - 4(n-2) quadradinhos.
> 
> A probabilidade de ocorrer (a) é:
> p1 = 4/n^2 * 2/(n^2 - 1)
> 
> A probabilidade de ocorrer (b) é:
> p2 = 4(n-2)/n^2 * 3/(n^2 - 1)
> 
> A probabilidade de ocorrer (c) é:
> p3 = [n^2 - 4 - 4(n-2)]/n^2 * 4/(n^2 - 1)
> 
> E, se não errei até agora, a probabilidade pedida é:
> 
> P = p1 + p2 + p3
> 
> 
> O que vocês acham?
> 
> 
> Obrigado,
> 
> Rafael de A. Sampaio
> 
> =
> Instruções para entrar na lista, sair da lista e usar a lista em
> http://www.mat.puc-rio.br/~nicolau/olimp/obm-l.html
> =
> 


=
Instruções para entrar na lista, sair da lista e usar a lista em
http://www.mat.puc-rio.br/~nicolau/olimp/obm-l.html
=


Re: [obm-l] Re: [obm-l] Princípio de Dirichlet

2004-05-09 Por tôpico Claudio Buffara
on 09.05.04 01:38, Ricardo Bittencourt at [EMAIL PROTECTED] wrote:

> Fabiano Sant'Ana wrote:
> 
>> como um simples ponto poderá possuir Duas cores?
> 
> Desenha um círculo no papel, aí pra cada ponto
> do círculo você pinta de azul por cima da folha, e de
> vermelho na parte de baixo. Pronto, agora um simples
> ponto tem duas cores!
> 
> Eu aqui estou trabalhando profissionalmente
> com coisa muito mais bizarra, que é ponto orientado
> (na verdade, vetor 2D degenerado cuja magnitude foi
> pra zero, mas preservou a direção e sentido).
> 
Acho que o enunciado quis dizer que, a cada ponto do circulo, associa-se uma
dentre duas cores, ou seja, define-se uma funcao F: C -> {azul,vermelho},
onde C eh o tal circulo.

[]s,
Claudio.


=
Instruções para entrar na lista, sair da lista e usar a lista em
http://www.mat.puc-rio.br/~nicolau/olimp/obm-l.html
=


RE: [obm-l] cococolegio navalvalval

2004-05-09 Por tôpico Rogério Moraes de Carvalho
Olá pessoal,

Vamos tentar prestar mais atenção na hora de copiar os enunciados
das questões, afinal de contas muitas pessoas estão gastando os seus tempos
para resolvê-las.

Vamos à resolução desta questão.

RESOLUÇÃO POSSÍVEL:

A idéia é tentar colocar a expressão 1/M^3 + 1/N^3 em função da soma das
raízes S = M + N = -b/a = 10 e do produtos das raízes P = M.N = c/a = 1.

Seja E = 1/M^3 + 1/N^3, então:
E = (M^3 + N^3)/(M^3.N^3)
E = [(M + N).(M^2 - M.N + N^2]/[(M.N)^3]
E = [(M + N).(M + N)^2 - 3.M.N]/[(M.N)^3]
E = [S.(S^2 - 3.P)]/(P^3)
E = [10.(10^2 - 3.1)]/(1^3)
E = 10.(100 - 3)
E = 970

Resposta: Letra A

Rogério Moraes de Carvalho
Consultor e Instrutor de Tecnologias da Informação
[EMAIL PROTECTED]


-Original Message-
From: [EMAIL PROTECTED] [mailto:[EMAIL PROTECTED] On
Behalf Of Pacini bores
Sent: sábado, 8 de maio de 2004 18:49
To: [EMAIL PROTECTED]
Subject: Re: [obm-l] cococolegio navalvalval


Olá  Leandro ,

O enunciado  correto eh :
  Sendo M e N as raízes da equação X^2-10X+1=0 , o
valor da expressão  1/M^3 + 1/N^3 ,  ok ?

[]´s Pacini



> COLÉGIO NAVAL (1989)
>
>   Sendo M e N as raízes da equação X^2-10X+1=0 , o
>valor da expressão  1/M^3 + 1/N^2 é :
>
>  (A) 970
>  (B) 950
>  (C) 920
>  (D) 900
>  (E) 870
>===
>  ,



=
Instruções para entrar na lista, sair da lista e usar a lista em
http://www.mat.puc-rio.br/~nicolau/olimp/obm-l.html
=




=
Instruções para entrar na lista, sair da lista e usar a lista em
http://www.mat.puc-rio.br/~nicolau/olimp/obm-l.html
=


Re: [obm-l] Calcular a soma

2004-05-09 Por tôpico Augusto Cesar de Oliveira Morgado
1/senx = -cotx + cot(x/2)

==
Mensagem  enviada  pelo  CIP  WebMAIL  - Nova Geração - v. 2.1
CentroIn Internet Provider  http://www.centroin.com.br
Tel: (21) 2542-4849, (21) 2295-3331Fax: (21) 2295-2978
Empresa 100% Brasileira - Desde 1992 prestando servicos online


-- Original Message ---
From: [EMAIL PROTECTED]
To: [EMAIL PROTECTED]
Sent: Sun, 9 May 2004 06:15:16 -0300
Subject: [obm-l] Calcular a soma

> Por favor alguém tem alguma idéia para calcular esta soma: 
> S=1/senX + 1/sen2X + 1/sen4X + 1/sen8X + ... + 1/sen(2^n)X
> 
> _
> Voce quer um iGMail protegido contra vírus e spams?
> Clique aqui: http://www.igmailseguro.ig.com.br
> Ofertas imperdíveis! Link: http://www.americanas.com.br/ig/
> 
> =
> Instruções para entrar na lista, sair da lista e usar a lista em
> http://www.mat.puc-rio.br/~nicolau/olimp/obm-l.html
> =
--- End of Original Message ---

=
Instruções para entrar na lista, sair da lista e usar a lista em
http://www.mat.puc-rio.br/~nicolau/olimp/obm-l.html
=


[obm-l] Calcular a soma

2004-05-09 Por tôpico valeriomoura
Por favor alguém tem alguma idéia para calcular esta soma: 
S=1/senX + 1/sen2X + 1/sen4X + 1/sen8X + ... + 1/sen(2^n)X 

_
Voce quer um iGMail protegido contra vírus e spams?
Clique aqui: http://www.igmailseguro.ig.com.br
Ofertas imperdíveis! Link: http://www.americanas.com.br/ig/

=
Instruções para entrar na lista, sair da lista e usar a lista em
http://www.mat.puc-rio.br/~nicolau/olimp/obm-l.html
=


[obm-l] Probabilidade e quadradinhos

2004-05-09 Por tôpico Rafael
Pessoal,

Em outra lista, vi um problema interessante:

Uma folha quadrada de papel quadriculado contém n^2 quadradinhos (n >= 2).
Escolhendo-se, ao acaso, dois quadradinhos distintos, qual é a probabilidade
de que eles tenham um lado comum?

..


Há três "tipos" de quadradinhos: a) aqueles que estão nos cantos da folha e
possuem apenas dois outros quadradinhos adjacentes; b) aqueles que estão ao
longo das extremidades da folha e possuem apenas três quadradinhos vizinhos;
e c) aqueles que não estão em qualquer uma das posições anteriores da folha
e possuem quatro quadradinhos vizinhos.

Qualquer que seja n >= 2, teremos para:

a) 4 quadradinhos;
b) 4(n-2) quadradinhos;
c) n^2 - 4 - 4(n-2) quadradinhos.

A probabilidade de ocorrer (a) é:
p1 = 4/n^2 * 2/(n^2 - 1)

A probabilidade de ocorrer (b) é:
p2 = 4(n-2)/n^2 * 3/(n^2 - 1)

A probabilidade de ocorrer (c) é:
p3 = [n^2 - 4 - 4(n-2)]/n^2 * 4/(n^2 - 1)

E, se não errei até agora, a probabilidade pedida é:

P = p1 + p2 + p3


O que vocês acham?


Obrigado,

Rafael de A. Sampaio

=
Instruções para entrar na lista, sair da lista e usar a lista em
http://www.mat.puc-rio.br/~nicolau/olimp/obm-l.html
=